You are on page 1of 106

Contents

NUMERICAL APTITUDE
1. Number System .............................................................................................................................. 3
2. Fractions ....................................................................................................................................... 18
3. Indices and Surds ......................................................................................................................... 32
4. Square Root & Cube Root ............................................................................................................. 48
5. Simplification ................................................................................................................................. 59
6. HCF & LCM................................................................................................................................... 69
7. Orders of Magnitude ...................................................................................................................... 82
8. Unitary Method .............................................................................................................................. 86
9. Problemon Ages ........................................................................................................................... 95
10. Average ......................................................................................................................................... 99
11. Percentage ................................................................................................................................... 114
12. Profit and Loss ............................................................................................................................. 130
13. Ratio & Proportion ........................................................................................................................ 147
14. Partnership ................................................................................................................................... 160
15. Alligation or Mixture ...................................................................................................................... 170
16. Time and Work ............................................................................................................................. 179
17. Pipes and Cisterns ....................................................................................................................... 199
18. Time & Distance ........................................................................................................................... 204
19. Problems on Trains ....................................................................................................................... 229
20. Boats and Streams ....................................................................................................................... 235
21. Simple Interest .............................................................................................................................. 238
22. Compound Interest ....................................................................................................................... 255
23. Area of Plane Figures ................................................................................................................... 273
24. Volume and Surface Area of Solid Figures .................................................................................... 297
25. Clocks and Calendar .................................................................................................................... 316
26. Sequences & Series..................................................................................................................... 327
DATA INTERPRETION & DATA SUFFICIENCY
1. Introduction to Data Interpretation .................................................................................................... 3
2. Approaches to Data Interpretation ................................................................................................... 6
3. Table Chart ...................................................................................................................................... 8
4. Pie Chart ....................................................................................................................................... 53
5. Bar Chart ...................................................................................................................................... 83
6. Line Graph.................................................................................................................................... 111
7. Mix Diagrams ............................................................................................................................... 145
REASONING
COMMERCIALREASONING
1. Analogy ........................................................................................................................................... 1
2. Classification ................................................................................................................................. 13
3. Series ............................................................................................................................................ 24
4. Coding-Decoding........................................................................................................................... 44
5. Blood Relations ............................................................................................................................. 61
6. Direction Sense Test ...................................................................................................................... 76
7. Logical Venn Diagrams .................................................................................................................. 96
8. Alphabet Test ................................................................................................................................ 107
9. Sitting Arrangements .................................................................................................................... 119
10. Mathematical Operations .............................................................................................................. 132
11. Arithmetical Reasoning ................................................................................................................. 147
12. Inserting the Missing Character .................................................................................................... 156
13. Number, Ranking and Time Sequence Test ................................................................................. 168
14. Syllogism...................................................................................................................................... 181
15. Logical Sequence of WOrds ......................................................................................................... 205
16. Alpha-numeric Sequence Test ...................................................................................................... 207
17. Puzzle Test ................................................................................................................................... 212
ANALYTICALREASONING
1. Statement & Conclusions ................................................................................................................ 3
2. Statement & Arguments................................................................................................................... 9
3. Statement & Assumptions ............................................................................................................. 17
4. Cause and Effect ........................................................................................................................... 25
5. Courses of Action.......................................................................................................................... 27
6. Inference ....................................................................................................................................... 33
NON-VERBALREASONING
1. Mirror Image .................................................................................................................................... 3
2. Water Image .................................................................................................................................... 5
3. Paper Folding .................................................................................................................................. 7
4. Paper Cutting................................................................................................................................... 9
5. Counting the Number of Figures .................................................................................................... 13
6. Dice Problems ............................................................................................................................... 15
7. Five-Figures Series ....................................................................................................................... 17
8. Missing Figure Series .................................................................................................................... 19
9. Analogy ......................................................................................................................................... 21
10. Classification ................................................................................................................................. 25
GENERAL ENGLISH
SECTIONI
1. Parts of Speech /Sentence ..................................................................................................... 3-30
SECTIONII
2. English Language Comprehension ..................................................................................... 31-65
SECTIONIII
1. Common Error ............................................................................................................................... 69
2. Sentence Correction...................................................................................................................... 88
3. Synonyms & Antonyms ................................................................................................................. 96
4. Idioms & Phrases ......................................................................................................................... 103
5. Phrasal Verb ................................................................................................................................. 123
6. Sentence Arrangement ................................................................................................................. 132
7. Foreign Words & Phrases ............................................................................................................ 138
9. Cloze Test .................................................................................................................................... 143
10. Commonly Misspelled English Words ........................................................................................... 151
COMPUTER & MARKETING
COMPUTERKNOWLEDGE
1. Fundamentals of Computer .............................................................................................................. 4
2. Components of Computer ................................................................................................................ 7
3. CPU ................................................................................................................................................ 8
4. Input Devices ................................................................................................................................ 10
5. Output Devices ............................................................................................................................. 13
6. Memory ......................................................................................................................................... 17
7. RAM & ROM ................................................................................................................................. 19
8. Motherboard .................................................................................................................................. 22
9. Memory Units ................................................................................................................................ 23
10. Hardware & Software .................................................................................................................... 26
11. Networking .................................................................................................................................... 28
12. Operating System ......................................................................................................................... 30
13. Internet & Intranet .......................................................................................................................... 32
14. Keyboard Shortcuts ....................................................................................................................... 33
15. Computer & Internet Glossary ....................................................................................................... 38
Multiple-Choice Questions ............................................................................................................. 46
MARKETINGAPTITUDE
1. Marketing: Nature and Scope ........................................................................................................ 69
2. Marketing Environment .................................................................................................................. 72
3. Marketing Mix ................................................................................................................................ 74
4. Product Life Cycle ......................................................................................................................... 77
5. Consumer Behaviour ..................................................................................................................... 79
6. Market Segmentation..................................................................................................................... 81
7. Marketing Channels ....................................................................................................................... 83
8. Middlemen in Marketing ................................................................................................................. 85
9. Sales Promotion ............................................................................................................................ 85
10. Advertising .................................................................................................................................... 87
11. Branding and Packaging ................................................................................................................ 89
12. Sales Forecasting ......................................................................................................................... 89
13. Marketing in Banking Sector .......................................................................................................... 91
Multiple Choice Questions ............................................................................................................. 95
Numerical Aptitude Miscellany
2013 bankpoclerk.com
In Indian system, numbers are expressed by means of symbols 0, 1, 2, 3, 4, 5, 6, 7, 8, 9, called digits. Here, 0 is called
insignificant digit whereas 1, 2, 3, 4, 5, 6, 7, 8, 9 are called significant digits. We can express a number in two ways.
Notation: Representing a number in figures is known as notation as 350.
Numeration: Representing a number in words is known as numeration as Five hundred and forty five.
Place Value (Indian)
Crore Lakh Thousand Unit
Ten Crore Crore Ten Lakhs Lakh Ten Thousands Thousand Hundred Tens One
100000000 10000000 1000000 100000 10000 1000 100 10 1
10
8
10
7
10
6
10
5
10
4
10
3
10
2
10
1
10
0
Place Value (International)
Million Thousand Unit
Hundred Ten Millions One Million Hundred Ten Thousand Hundred Tens One
Millions Thousands Thousands
100000000 10000000 1000000 100000 10000 1000 100 10 1
10
8
10
7
10
6
10
5
10
4
10
3
10
2
10
1
10
0
Face Value and Place Value of a Digit
Face Value: It is the value of the digit itself eg, in 3452, face value of 4 is four, face value of 2 is two.
Place Value: It is the face value of the digit multiplied by the place value at which it is situated eg, in 2586, place value
of 5 is 5 10
2
= 500.
Number Categories
Natural Numbers (N): If N is the set of natural numbers, then we write N = {1, 2, 3, 4, 5, 6,}
The smallest natural number is 1.
Whole Numbers (W): If W is the set of whole numbers, then we write W = {0, 1, 2, 3, 4, 5,}
The smallest whole number is 0.
Integers (I): If I is the set of integers, then we write I = { 3, 2, 1, 0, 1, 2, 3, }
1
Number System

Click Here For Hard Copy of this Study Materials:
http://bankpoclerk.com/community/study-kit/ibps-po-capsule
For Any Guidance Call Our Expert at +91 8800734161
Numerical Aptitude Number System
2013 bankpoclerk.com
Rational Numbers: Any number which can be expressed in the form of p/q, where p and q are both integers and q #
0 are called rational numbers.
eg,
3 7
, ,5, 2
2 9


There exists infinite number of rational numbers between any two rational numbers.
Irrational Numbers Non-recurring and non-terminating decimals are called irrational numbers. These
numbers cannot be expressed in the form of
p
q
.
eg, 3, 5, 29,
Real Numbers: Real number includes both rational and irrational numbers.
Basic Rules on Natural Numbers
1. One digit numbers are from 1 to 9. There are 9 one digit numbers. ie, 9 10
0
.
2. Two digit numbers are from 10 to 99. There, are 90 two digit numbers. ie, 9 10.
3. Three digit numbers are from 100 to 199. There are 900 three digit numbers ie, 9 10
2
.
In general the number of n digit numbers are 9 10
(n1)
4. Sum of the first n, natural numbers ie, 1 + 2 + 3 + 4 + + n =
( ) 1
2
n n +
5. Sum of the squares of the first n natural numbers ie. 1
2
+ 2
2
+ 3
2
+ 4
2
+ + n
2
=
( )( ) 1 2 1
6
n n n + +
6. Sum of the cubes of the first n natural number ie, 1
3
+ 2
3
+ 3
3
+ + n
3
=
( )
1 +
1
1
]
2
1
2
n n
Example: What is the value of 51 + 52 + 53 + + 100 ?
Solution. 51 + 52 + 33 + ... + 100 = (1 + 3 + + 100) (1 + 2 + 3 + ... + 50)
=
100 101 50 51
2 2

= 5050 1275 = 3775


Different Types of Numbers
Even Numbers: Numbers which are exactly divisible by 2 are called even numbers.
eg, 4, 2, 0, 2, 4
Sum of first n even numbers = n (n + 1)
Odd Numbers: Numbers which are not exactly divisible by 2 are called odd numbers.
eg, 5, 3, 1, 0, 1, 3, 5
Sum of first n odd numbers = n
2
Prime Numbers: Numbers which are divisible by one and itself only are called prime numbers.
For Any Guidance Call Our Expert at +91 8800734161

Click Here For Hard Copy of this Study Materials:
http://bankpoclerk.com/community/study-kit/ibps-po-capsule
Numerical Aptitude Number System
2013 bankpoclerk.com
eg, 2, 3, 5, 7, 11
2 is the only even prime number.
1 is not a prime number because it has two equal factors.
Every prime number greater than 3 can be written in the form of (6K + 1) or (6K 1) where K is an integer. There are
15 prime numbers between 1 and 50 and l0 prime numbers between 50 and 100.
Relative Prime Numbers: Two numbers are said to be relatively prime if they do not have any common factor other
than 1.
eg, (3, 5), (4, 7), (11, 15), (15, 4)
Twin Primes: Two prime numbers which differ by 2 are called twin primes.
eg, (3, 5), (5, 7), (11, 13),
Composite Numbers Numbers which are not prime are called composite numbers.
eg, 4, 6, 9, 15,
1 is neither prime nor composite.
Perfect Number: A number is said to be a perfect number, if the sum of all its factors excluding itself is equal to the
number itself. eg, Factors of 6 are 1, 2, 3 and 6.
Sum of factors excluding 6 = 1 + 2 + 3 = 6.
6 is a perfect number.
Other examples of perfect numbers are 28, 496, 8128 etc.
Rules for Divisibility
Divisibility by 2: A number is divisible by 2 when the digit at ones place is 0, 2, 4, 6 or 8.
eg, 3582, 460, 28, 352, ....
Divisibility by 3: A number is divisible by 3 when sum of all digits of a number is a multiple of 3.
eg, 453 = 4 + 5 + 3 = 12.
12 is divisible by 3 so, 453 is also divisible by 3.
Divisibility by 4: A number is divisible by 4, if the number formed with its last two digits is divisible by 4. eg, if we take
the number 45024, the last two digits form 24. Since, the number 24 is divisible by 4, the number 45024 is also divisible by 4.
Divisibility by 5: A number is divisible by 5 if its last digit is 0 or 5.
eg, 10, 25, 60
Divisibility by 6: A number is divisible by 6, if it is divisible both by 2 and 3.
eg, 48, 24, 108
Divisibility by 7: A number is divisible by 7 when the difference between twice the digit at ones place and the number
formed by other digits is either zero or a multiple of 7.
eg, 658
65 2 8 = 65 16 = 49
As 49 is divisible by 7 the number 658 is also divisible by 7.
Divisibility by 8: A number is divisible by 8, if the number formed by the last 3 digits of the number is divisible by 8. eg,
if we take the number 57832, the last three digits form 832. Since, the number 832 is divisible by 8, the number 57832 is also
divisible by 8.
Divisibility by 9: A number is divisible by 9, if the sum of all the digits of a number is a multiple of 9.
Numerical Aptitude Number System
For Any Guidance Call Our Expert at +91 8800734161

Click Here For Hard Copy of this Study Materials:
http://bankpoclerk.com/community/study-kit/ibps-po-capsule
Numerical Aptitude Number System
2013 bankpoclerk.com
eg, 684 = 6 + 8 + 4 = 18.
18 is divisible by 9 so, 684 is also divisible by 9.
Divisibility by 10: A number is divisible by 10, if its last digit is 0. eg, 20, 180, 350,.
Divisibility by 11: A number is divisible by 1) When the difference between the sum of its digits in odd places and in
even places is either 0 or a multiple of 11.
eg, 30426
3 + 4 + 6 = 13
0 + 2 = 2
13 2 = 11
As the difference is a multiple of 11 the number 30426 is also divisible by 11.
Smart Facts
If p and q are co-primes and both are factors of a number K, then their product p q will also be a factor of k. eg,
Factors of 24 are 1, 2, 3, 4, 6, 8, 12 and 24 prime factors of 24 are 2 and 3, which are co-prime also. Product of
2 3 = 6, 6 is also a factor of 24.
If p divides q and r, then p also divides their sum or difference. eg, 4 divides 12 and 20. Sum of 12 and 20 is 32
which is divisible by 4. Difference of 20 and 12 is 8 which is divisible by 4.
If a number is divisible by another number, then it must be divisible by each of the factors of that number. 48 is divisible
by 12. Factors of 12 are 1, 2, 3, 4, 6, 12. So, 48 is divisible by 2, 3, 4 and 6 also.
Division on Numbers
In a sum of division, we have four quantities.
They are (i) Dividend, (ii) Divisor, (iii) Quotient and (iv) Remainder. These quantities are connected by a relation.
(1) Dividend = Divisor Quotient + Remainder.
(2) Divisor = (Dividend Remainder) Quotient.
(3) Quotient = (Dividend Remainder) Divisor.
Example 2: In a sum of division, the quotient is 110, the remainder is 250, the divisor is equal to the sum of the quotient
and remainder. What is the dividend ?
Solution. Divisor = (110 + 250) = 360
Dividend = (360 110) + 250 = 39850
Hence, the dividend is 39850.
Example 3: Find the number of numbers upto 600 which are divisible by 14.
Solution. Divide 600 by 13, the quotient obtained is 46. Thus, there are 46 numbers less than 600 which are divisible by
14.
Factors and Multiples
Factor: A number which divides a given number exactly is called a factor of the given number,
eg, 24 = 1 24, 2 12, 3 8, 4 6
Thus, 1, 2, 3, 4, 6, 8, 12 and 24 are factors of 24.
1 is a factor of every number
A number is a factor of itself
Numerical Aptitude Number System
For Any Guidance Call Our Expert at +91 8800734161

Click Here For Hard Copy of this Study Materials:
http://bankpoclerk.com/community/study-kit/ibps-po-capsule
Numerical Aptitude Number System
The smallest factor of a given number is 1 and the greatest factor is the number itself.
If a number is divided by any of its factors, the remainder is always zero.
Every factor of a number is either less than or at the most equal to the given number.
Number of factors of a number are finite.
Number of Factors of a Number: If N is a composite number such that N = a
m
b
n
c
o
... where a, b, c ... are prime
factors of N and m, n, o ... are positive integers, then the number of factors of N is given by the expression (m + 1) (n + 1)
(o + 1)
Example 4: Find the number of factors that 224 has.
Solution. 224 = 2
5
7
1
Hence, 224 has (5 + 1) (1 + 1) = 6 2 = 12 factors.
Multiple: A multiple of a number is a number obtained by multiplying it by a natural number eg,
Multiples of 5 are 5, 10, 15, 20
Multiples of 12 are 12, 24, 36, 48
Every number is a multiple of 1.
The smallest multiple of a number is the number itself.
We cannot find the greatest multiple of a number.
Number of multiples of a number are infinite.

1. Evaluate:
( )
9 3 5 5 4 10
3 5 24 2


(1) 9/10 (2) 8/17
(3) 16/19 (4) 4/7
(5) None of these
2. The sum of three consecutive natural numbers each
divisible by 3 is 72. What is the largest among them?
(1) 25 (2) 26
(3) 27 (4) 30
(5) None of these
3. 55% of a number is more than one-third of that number
by 52. What is two-fifth of that number?
(1) 96 (2) 240
(3) 144 (4) 142
(5) None of these
4. The digits of a two-digit number are in the ratio of 2 : 3
and the number obtained by interchanging the digits is
bigger than the original number by 27. What is the
original number?
(1) 63 (2) 48
(3) 96 (4) 69
(5) None of these
Numerical Aptitude Number System
For Any Guidance Call Our Expert at +91 8800734161

Click Here For Hard Copy of this Study Materials:
http://bankpoclerk.com/community/study-kit/ibps-po-capsule
ANSWERS
1. (3) 2. (3) 3. (1) 4. (4)

EXPLANATIONS
1.
( )


9 3 5 5 4 10
3 5 2 4 2
=
( ) ( )
( )


9 2 5 4 10
3 5 2 2
=


18 2
15 4
=

16
19
2. 3x + (3x + 3) + (3x + 6) = 72
9x + 9 = 72 = 9x = 72 9
or x =
63
9
= 7
The largest of them is 27.
3. Let the number be x.
55
100
x
=
1
52
3
x +
13
60
x
= 52 x = 240

2
5
x
=
2
240
5
= 96
4. Let the number be 10x + y
x : y = 2 : 3 (i)
(10y + x) (10x + y) = 27 ...(ii)
Numerical Aptitude Miscellany
2013 bankpoclerk.com
A fraction is a part of the whole (object, thing, region). It forms the part of basic aptitude of a person to have and idea of
the parts of a population, group or territory. Aspirants must have a feel of fractional thinking. eg,
5
12
, here 12 is the number
of equal part into which the whole has been divided, is called denominator and 5 is the number of equal parts which have
been taken out, is called numerator.
Example1: Name the numerator of
3
7
and denominator of
5
.
13
Solution: Numerator of
3
7
is 3.
Denominator of
5
13
is 13.
Lowest Term of a Fraction
Dividing the numerator and denominator by the highest common element (or number) in them, we get the fraction in its
lowest form.
eg, To find the fraction
6
14
in lowest form Since 2 is highest common element in numerator 6 and denominator 14 so
dividing them by 2, we get
3
.
7
Which is the lowest formof
6
.
14
Equivalent Fractions
If numerator and denominator of any fraction are multiplied by the same number then all resulting fractions are called
equivalent fractions.
eg,
1 2 3 4
, , ,
2 4 6 8
all are equivalent fractions but
1
2
is the lowest form.
2
Fractions
Numerical Aptitude Number System
For Any Guidance Call Our Expert at +91 8800734161

Click Here For Hard Copy of this Study Materials:
http://bankpoclerk.com/community/study-kit/ibps-po-capsule
Numerical Aptitude Fractions
2013 bankpoclerk.com
Example 2: Find the equivalent fractions of
2
5
having numerator 6.
Solution: We know that 2 3 = 6. This means we need to multiply both the numerator and denominator by 3 to get the
equivalent fraction.
Hence, required equivalent fraction

2 2 3 6
5 5 3 15
Addition and Subtraction of Fractions
Here two cases arise as denominators of the fraction are same or not.
Case I: When denominators of the two fractions are the same then we write denominator once and add (or subtract) the
numerators.
eg,
+
2 3
7 7
=
5
7
Case II: If denominators are different, we need to find a common denominator that both denominators will divide into.
eg,
+
1 3
6 8
We can write,
1
6
=
2 3
,
12 18
=
4
24
3
8
=
6
16
=
9
24

+
1 3
6 8
=
+
4 9
24 24
=
13
24
Example 3: Calculate

1 3
2 7
Solution.
1
2
=

1 7
2 7
=
7
14
and
3
7
=

3 2
7 2
=
6
14


1 3
2 7
=

7 6
14 14
=
1
14
Multiplication and Division of Fractions
To multiply fractions, the numerators are multiplied together and denominators are multiplied together.
eg,

1 3
6 8
=

1 3
6 8
=
3
48
=
1
16
In division of fraction, the numerator of first fraction is multiplied by the denominator of second fraction and gives the
numerators. Also denominator of first fraction is multiplied by the numerator of second fraction and gives the denominator.
eg,

1 3
6 8
becomes

1 8
6 3
=
8
18
=
4
9
Numerical Aptitude Number System
For Any Guidance Call Our Expert at +91 8800734161

Click Here For Hard Copy of this Study Materials:
http://bankpoclerk.com/community/study-kit/ibps-po-capsule
Numerical Aptitude Fractions
2013 bankpoclerk.com
Proper and Improper Fractions
The fractions in which the number in numerator is less than that of denominator, are called proper fractions. Also if the
number in numerator is greater than that of denominator, then the fractions are called improper fractions.
eg,
4
3
is an improper fraction while
3
4
is a proper fraction.
Mixed Numbers
A mixed number is that, which contains both a whole number and a fraction.
eg,
7 1 5
4 ,3 ,6
12 4 8
are mixed numbers.
Example 4: Which of the following are proper and improper fractions?
(1)
7
9
(2)
6
5
(3)
12
7
(4)
5
13
Solution. (1) and (4) are proper fractions as numerator is less than denominator.
Also, (2) and (3) are improper fractions as numerator is greater than denominator.
Example 5: Are
7
13
and
5
4
6
mixed number?
Solution.
7
13
is only a proper fraction as it does not contain any whole number, while
5
4
6
is a mixed number as it
contains 4 as a whole number and
5
6
as a fraction.
Decimal Fractions
The fractions in which denominators has the power of 10 are called decimal fractions.
eg, 0.25
=
25
100
=
1
4
= one quarter
0.1 = point one =
1
10
= one-tenth.
For converting a decimal fraction into simple fraction, we write the numerator without point and in the denominator,
we write 1 and put the number of zeros as many times as number of digits after the point in the given decimal
fraction
eg, 0.037 =
37
,0.1257
1000
=
1257
10000
Example 6: Convert the each of the following decimal fractions into simple fractions.
(1) 5.76 (2) 0.023 (3) 257.5
Solution.
(1) 5.76 =
576
100
=
288
50
=
144
25
Numerical Aptitude Number System
For Any Guidance Call Our Expert at +91 8800734161

Click Here For Hard Copy of this Study Materials:
http://bankpoclerk.com/community/study-kit/ibps-po-capsule
Numerical Aptitude Fractions
2013 bankpoclerk.com
(2) 0.023 =
23
1000
(3) 257.5 =
2575
10
=
515
2
Addition or Subtraction of Decimal Fractions
In the addition or subtraction of decimal fractions, we write the decimal fractions in such a way that all the decimal points
are in the same straight line then these numbers can be added or subtracted in simple manner.
Example 7: Solve 0.68 + 0.062 + 0.20
Solution. 0.680
+0.062
+0.200
0.942
Multiplication of Decimal Fractions
To multiply by multiplies (powers) of 10 the decimal point is moved to the right by the respective number of zero.
Example 8: 0.75 10 = ?
Solution. 0.75 10 = 7.5 (The decimal point is shifted to right by one place)
To multiply decimals by number other than 10. We ignore the decimal point and multiply them in simple manner and at last
put the points after the number of digits (from right) corresponding to the given problem.
Example 9: Multiply 8 and 10.24
Solution. First we multiply 8 and 1024
8 1024 = 8192
Now, 8 10.24 = 81.92 (We put decimal points after two digits from right as
in given question).
Example 10: 12.4 1.62
Solution. We know that
124 162 = 20088
12.4 1.62 = 20.088
Division of Decimal Fractions
Division of decimal numbers is the reverse of the multiplication case ie, we move the decimal point to the left while
dividing by multiplies of 10.
Example 11: 25.75 10 = ?
Solution. 25.75 10 = 2.575
When a decimal number is divided by an integer, then at first divide the number ignoring the decimal point and at last put
the decimal after the number of digits (from right) according to the given problem.
Example 12: Divide 0.0221 by 17
Solution. First we divide 221 by 17 ie, without decimal 221 17 = 13
Now, we put decimal according as in the given problem
0.0221 17 = 0.0013
Numerical Aptitude Number System
For Any Guidance Call Our Expert at +91 8800734161

Click Here For Hard Copy of this Study Materials:
http://bankpoclerk.com/community/study-kit/ibps-po-capsule
Numerical Aptitude Fractions
2013 bankpoclerk.com
If divisor and dividend both are decimal numbers then first we convert them in simple fraction by putting number of zero
in the denominator of both. Then divide by the above manner.
Example 13: Divide by 0.0256 by 0.016
Solution.
0.0256
0.016
=

256 1000
16 10000
=
256
160
=
25.6
16
=1.6
Example 14: Divide 70.5 by 0.25
Solution.
70.5
0.25
=

705 100
25 10
=
7050
25
= 282
To Find HCF and LCM of Decimal Fractions
First we make the decimal digits of the given decimal numbers, same by putting some number of zero if necessary. Then
find HCF or LCM ignoring decimals. And at last put the decimal according to the given numbers.
Example 15: Determine HCF and LCM of 0.27, 1.8 and 0.036.
Solution. Given numbers are 0.27, 1.8 and 0.036.
or 0.270, 1.800 and 0.036.
These numbers without decimals are 270, 1800 and 36.
Now, HCF of 270, 1800 and 36 = 18
HCF of 0.270, 1.800 and 0.036 = 0.018
LCM of 270, 1800 and 36 = 18 5 2 3 10 = 5400
18 270 1800 36
5 15 100 2
2 3 20 2
3 10 1
LCMof 0.270, 1.800 and 0.36 = 5.400 or 5.4
Terminating and Non-Terminating Recurring Decimals
If decimal expression of any fraction is terminated then fraction is called terminating.
as
5
16
= 0.3125
But if we take example 33 26, then
26) 33 (1.2692307
26
70
52
180 A
156
240
234
60
Numerical Aptitude Number System
For Any Guidance Call Our Expert at +91 8800734161

Click Here For Hard Copy of this Study Materials:
http://bankpoclerk.com/community/study-kit/ibps-po-capsule
Numerical Aptitude Fractions
2013 bankpoclerk.com
52
80
78
200
182
180 B
In this division, we see that remainder at the stages A and B are the same. In the continued process of division by 26, the
digits 6, 9, 2, 3, 0, 7 in the quotient will repeat onwards.

33
26
= 1.2692307692307...
This process of division is non terminating. Therefore, such decimal expressions are called nonterminating repeating
(recurring) decimals.
In repeating digit, we put () bar.
ie,
33
26
=
1.2692307
Example 16: Write the following fractions in decimal form and till that these are terminating or nonterminating recurring.
(1)
2
3
(2)
4
5
(3)
3
11
(4)
17
90
Solution.
(1)
2
3
= 0.6666.... =
0.6
nonterminating recurring
(2)
4
5
= 0.8 terminating
(3)
3
11
= 0.272727... =
0.27
nonterminating recurring
(4)
17
90
= 0.1888...
0.18
nonterminating recurring
Non-Terminating, Non-Recurring Decimals
Every fraction can be put in the form of terminating or non-terminating recurring decimals ie, these decimal numbers can
be put in the form of
p
q
. These are called rational numbers. But some decimals numbers are there that cant be put in the form
of
p
q
, these are non-terminating, non-recurring decimals. Also these are called irrational numbers.
eg, 0.101001000100001...
To convert non-terminating recurring decimals into simple fraction
First write the non-terminating recurring decimal in bar notation. Then write the digit a in the denominators as many
times as number of digits recurring in the numerator. Also dont put decimal in the numerator.
Numerical Aptitude Number System
For Any Guidance Call Our Expert at +91 8800734161

Click Here For Hard Copy of this Study Materials:
http://bankpoclerk.com/community/study-kit/ibps-po-capsule
Numerical Aptitude Fractions
2013 bankpoclerk.com
Example 17: Convert the following in simple fraction
(i) 0.33333... (ii) 0.181818...
Solution.
(i) 0.33333... =
0.3
=
3
9
=
1
3
(ii) 0.181818... =
0.18
=
18
99
=
2
11
Mixed Recurring Decimals
A decimal fraction in which some digits are not repeated and some are repeated, called mixed recurring decimal.
How to Convert Mixed Recurring Decimal into a Simple Fraction?
First we subtract nonrepeated part from the number (without decimal) and put number 9 as many times as number of
recurring digits and also put the number 0 as many times odd number of non-recurring digits.
Example 18: Convert the following in simple fraction
(i)
0.18
(ii)
3.0072
Solution. (i)
0.18
= 0.1888 (i)
Multiply both side by 10
10
0.18
= 1.888 (ii)
Multiply both side by 100
100
0.18
= 18.888 (iii)
Substract (ii) from (iii) we have
90
0.18
= 18 1


0.18
=
17
90
(ii)
3.0072
= 3 +
0.0072
=
+
72
3
9900
=
+
2
3
275
=
2
3
275
Example 19: Arrange
2 13 4
, ,
3 15 5
and
15
16
in ascending order, ,
Solution. Here
2
3
= 0.67
13
15
= 0.86
4
5
= 0.80
15
16
= 0.94
Here, it is clear that 0.67 < 0.80 < 0.86 < 0.94

< < <


2 4 13 15
3 5 15 16
Numerical Aptitude Number System
For Any Guidance Call Our Expert at +91 8800734161

Click Here For Hard Copy of this Study Materials:
http://bankpoclerk.com/community/study-kit/ibps-po-capsule
Numerical Aptitude Fractions
2013 bankpoclerk.com
Example 20: Arrange
7 8 5
, ,
9 11 13
and
2
7
in descending order..
Solution. Here
7
9
= 0.77,
8
11
= 0.72,
5
13
= 0.38
2
7
= 0.28
Here it is clear that
> > > 0.77 0.72 0.38 0.28

> > >


7 8 5 2
9 11 13 7

1. If 2025 = 45, then the value of


0.00002025 0.002025 2025 + + +
20.25
=
(1) 49.95 (2) 49.5495
(3) 4.9995 (4) 499.95
(5) None of these
2. If 15 = 3.88, then the value of
5
3
is:
(1) 1.39 (2) 1.29
(3) 1.89 (4) 1.63
(5) None of these
3. If 2805 2.55 = 1100, then 280.5 25.5 is:
(1) 111 (2) 1.1
(3) 0.11 (4) 11
(5) None of these
4. The value of 213 + 2.013 + 0.213 + 2.0013 is:
(1) 217.2273 (2) 21.8893
(3) 217.32 (4) 3.217.32
(5) None of these
Numerical Aptitude Number System
For Any Guidance Call Our Expert at +91 8800734161

Click Here For Hard Copy of this Study Materials:
http://bankpoclerk.com/community/study-kit/ibps-po-capsule
ANSWERS
1. (2) 2. (2) 3. (4) 4. (1) 5. (1)
6. (4) 7. (1) 8. (1) 9. (1) 10. (4)
EXPLANATIONS
1. 2025 = 45
0.00002025 0.002025 2025 20.25 + + +
= 0.0045 + 0.045 + 45 + 4.5
= 49.5495
2. 15 = 3.88
5
3
=
5 3
3 3

=
15
3
=
3.88
3
= 1.29
3. 280.5 25.5 =
2805
2.55 10 10
=
1
1100
100

= 111
Numerical Aptitude I ndices and Surds
2013 bankpoclerk.com
-
In the expression x
n
, n is called the exponent or index and x is called the base and x
n
is read as x to the power of n or
x raised to the power n.
eg,
1. 3
6
= 3 3 3 3 3 3 = 729
4
3
= 4 4 4 = 64
The expression (x
m
)
n
is read as x raised to the power m whole raised to the power n.
2. (2
4
)
3
= (16)
3
= 2
12
The expression
n
m
x
is read as x raised to the power m power n.
3.
3
4
2
= 2
64
Hence, (x
m
)
n

n
m
x
Laws of Indices
1. a
m
a
n
= a
m + n
2.
m
n
a
a
= a
mn
(a 0)
3. (a
m
)
n
= a
mn
4. (ab)
m
= a
m
b
m
5.
_

,
m
a
b
=
m
m
a
b
(b 0) 6. a
0
= 1 if (a 0)
7. a
n
=
1
n
a
if (a 0) 8.
1
n
a
=
n
a
is called the n
th
root of a.
9.
m
n
a
=
n m
a
is called the n
th
root of a
m
.
10. When the bases of two numbers are equal, then their powers are also equal.
eg, If 3
n
= 3
2
, then, n = 2.
3
Indices and Surds
Numerical Aptitude I ndices and Surds
2013 bankpoclerk.com
11. When the powers of two numbers are equal, then their bases are equal.
eg, If a
3
= 4
3
, then a = 4.
Where a is a positive real number and n is a rational number.
-
If n is a positive integer and a is a positive rational number (a > 0), then
n
a
is called a surd of order n or nth order surd
if
n
a
is an irrational number. .
+ +
3 4
5, 7,2 7,2 7, 3 5, 3 5
1. Simple Surd: A surd which consists of a single term is called surd or monomial surd.
eg,
4
3, 3,
n
a
2. Mixed Surd: If a is a rational number and
b
is a surd, then
+ , a b a b
are called mixed surds.
eg,
+ 2 3,2 3
3. Compound Surd: A surd which is a sum or difference of two or more surds is called a compound surd.
eg,
+ 2 3,2 3
4. Similar Surds: If two surds are different multiples of the same surd, they are called similar surds otherwise they are
dissimilar surds.
eg,
2 3,4 3,5 3
are similar surds.
5 2,7 3,3 5
are dissimilar surds.
Laws of Radicals
(i)

n n
a b
=
n
ab
(ii)
n
n
a
b
=
n
a
b
(iii)
m n
a
=
mn
a
=
n m
a
(iv)
n p
a
=
p
n
a
=
mn pm
a
(v)
( )
n
n
a = a
Rationalisation of a Surd
If the product of two surds is rational, then each of the two surds is called a rationalising factor of the other. In general
if the surd is of type
+ a b
, then its rationalising factor is
a b
.
The rationalising factor of a surd is not unique. But it is always convenient to use the simplest of all rationalising.
Factors of a given Surd
(1) Rationalisig Factor of
1
a
=
a
(2) Rationalising Factor of
t
1
a b
= a m
b
(3) Rationalising Factor of
1
a b t
=
a b m
Numerical Aptitude Number System
For Any Guidance Call Our Expert at +91 8800734161

Click Here For Hard Copy of this Study Materials:
http://bankpoclerk.com/community/study-kit/ibps-po-capsule
Numerical Aptitude I ndices and Surds
2013 bankpoclerk.com
Smart Facts
1. The symbol
n
a
is called the radical sign.
2. In
n
a
, n is called the order of the surd.
3. In
n
a
, a is called the radicanl.
4. By
n
a
is a surd is understood that a is rational number and
n
a
is an irrational number. .
Comparison of Surds
Comparison of surds is possible only when they are of the same order. The radicals can be compared.
eg,
3 3
2 and 8 =
1 1
3 3
2 and8
and since 8 > 2
3 3
8 2 >
.
If the surds are of different order and different base, we first reduce them to the same order.
eg,
3 4
5and 4 are
1 1
3 4
5 and4
. The LCM of 4 and 3 is 12.

1
4
5
=
3
12
5
=
1
3
12
(5 )
=
12
125
1
3
4
=
4
12
4
=
1
4
12
(4 )
=
1
12
(256)
=
12
256
and 256 > 125
Hence,
3 4
4 5 >
Example 1: Simplify
(1)
3
7
(128)
(2) 2
25
2
35
2
20
2
40
Solution.
(1)
3
7
(128)
=
3
7
7
(2 )
= 2
3
= 8 (2) 2
25
2
35
2
20
2
40
= 2
60
2
60
= 0
Example 2. If a = 3 and b = 2, then find the values of
(1) a
a
+ b
b
(2)
1 1
a
a b
_
+

,
Solution. (1) a
a
+ b
b
= 3
3
+ 2
2
= 27 + 4 = 31 (2)
1 1
a
a b
_
+

,
=
3
1 1
3 2
_
+

,
=
3
5
6
_

,
=
125
216
Example 3. Evaluate
(1)
1
3
(64)

(2)
2
3
(125)

Solution.
(1)
1
3
(64)

=
1
3
1
(64)
=
1
3
3
1
(4 )
=
1
4
(2)
2
3
(125)

=
2
3
1
(125)
=
2
3
3
1
(5 )
=
2
1
5
=
1
25
Numerical Aptitude Number System
For Any Guidance Call Our Expert at +91 8800734161

Click Here For Hard Copy of this Study Materials:
http://bankpoclerk.com/community/study-kit/ibps-po-capsule
Numerical Aptitude I ndices and Surds
2013 bankpoclerk.com

1. The expression
( ) .85 .105 .024 .008
.022 .25 1.7
+ -

simplifies
to:
(1) 11 (2) 1.1
(3) 0.11 (4) .011
(5) None of these
2. The value of
16 1
36 4
+
is:
(1) 4/5 (2) 1/3
(3) 5/6 (4) 8/15
(5) None of these
3. A decimal number has 16 decimal places. The number
of decimal places in the square root of this number will
be:
(1) 7 (2) 4 (3) 8 (4) 16
(5) None of these
Numerical Aptitude Number System
For Any Guidance Call Our Expert at +91 8800734161

Click Here For Hard Copy of this Study Materials:
http://bankpoclerk.com/community/study-kit/ibps-po-capsule
EXPLANATIONS
1.
( ) +

0.85 0.105 0.024 0.008
0.022 0.25 1.7
=


0.85 0.121
0.022 0.25 1.7
=


85 121 10
22 25 17
= 11
2. +
16 1
36 4
= +
4 1
9 4
=
+ 16 9
36
=
25
36
=
5
6
3. The number of decimal places in the square root of a
number is half of the number of decimal places in that
number.
ANSWERS
1. (1) 2. (3) 3. (3)
Numerical Aptitude Square Root & Cube Root
2013 bankpoclerk.com
Square Root
The square root of a number is that number the product of which itself gives the given number, ie, the square root of 400
is 20, the square root of 625 is 25.
The process of finding the square root is called evaluation. The square root of a number is denoted by the symbol called
the radical sign. The expression
9
is read as root time, radical nine or the square root of nine.
How to Find the Square Root of an Integer?
(i) By the method of Prime Factors: When a given number is a perfect square, we resolve it into prime factors and
take the product of prime factors, choosing one out of every two.
Example 1: Find the square root of 4356.
Solution.
2 4356
2 2178
3 1089
3 363
11 121
11
4356 = 2 2 3 3 11 11 = 2
2
3
2
11
2
4356
= 2 3 11 = 66
Thus from the above example it is clear that in order to find the complete square root of a given number every prime
factor of that number should be repeated twice. Thus, we can make a number which is not a perfect square, a perfect
square by multiplying or dividing the number by those factors of it which are not contained in pairs.
Example 2: Find the least number by which 1800 can be multiplied or divided to make it a perfect square.
Solution. 1800 = 2 2 2 3 3 5 5
The least number by which the given number be multiplied or divided is 2.
4
Square Root & Cube Root
Numerical Aptitude Square Root & Cube Root
2013 bankpoclerk.com
(ii) By the method of Long Division: This method can be used when the number is large and the factors cannot be
determined easily. This method can also be used when we want to add a least number or to subtract a least number from a
given number so that the resulting number may give a perfect square of some number.
Example 3: Find the square root of 156816.
Solution.
3 156816 396
9
69 668
621
786 4716
4716

Explanations: Firstly, mark off the digits in pairs starting from the units digit. Each pair is called a period.
Now, 3
2
= 9 and 4
2
= 16. So we take 3
2
= 9 and on subtracting 9 from 15 we get 6 as remainder.
Now, bring down the next period ie,
Now, double the root figure already found which is 3 and write it to the left.
Now, from trial and error we find 69 9 = 621 which is closet and least to 668. So, place 9 to right of 6 changing it to 69.
We also put another 9 to the right of the quotient 3 making it 39. Now, we subtract 621 from 668. We get a remainder of 47.
Now, repeat the whole process till there is no period left over to be brought down.
So,
156816
= 396
To Find the Square Root of a Decimal
Example 4: Find the square root of 1.8225
Method 1:
1 1.8225 1.35
1
23 82
69
265 1325
1325

\
1.8225
= 1.35
Method 2:
1.8225
=
18225
10000
=
18225
10000
=
135
100
= 1.35
To Find the Square Root of a Fraction
Example 5: Find the square root of
13
1 .
36
Numerical Aptitude Number System
For Any Guidance Call Our Expert at +91 8800734161

Click Here For Hard Copy of this Study Materials:
http://bankpoclerk.com/community/study-kit/ibps-po-capsule
Numerical Aptitude Square Root & Cube Root
2013 bankpoclerk.com
Solution:
13
1
36
=
49
36
=
49
36
=
7
6
=
1
1
6
The square of a number other than unity is either a multiple of 4 or exceeds a multiple of 4 by 1.
A perfect square can never end with (1) an odd number or zeroes. (2) 2, 3, 7 and 8.
The square root of an integer is not always an integer ie,
3, 5, 11
are not integers.

ab
=
a b

a a
b
b

a b a b + +

a b a b
Example 6: Find the square root of 0.0016.
Solution.
0.0016
=
16
10000
=
16
10000
=
4
100
= 0.04
Example 7: Find the value of x if
25.6
x
= 8.
Solution. Given
25.6
x
= 8
25.6
x
= 64
64x = 25.6 x = 0.4
Example 8: Find the value of
5
upto three places of decimal.
Solution.
2 5.000000 2.236
4
42 100
84
443 1600
1329
4466 27100
26796

5
= 2.236
Example 9: If
5
= 2.236, find the value of
1
245 80 20
2

correct to three places of decimal.
Numerical Aptitude Number System
For Any Guidance Call Our Expert at +91 8800734161

Click Here For Hard Copy of this Study Materials:
http://bankpoclerk.com/community/study-kit/ibps-po-capsule
Numerical Aptitude Square Root & Cube Root
2013 bankpoclerk.com
Solution.
1
245 80 20
2

=
16 5
49 5 4 5
2


=
4 5
7 5 2 5
2
=
7 5 2 5 2 5
=
3 5
= 3 2.236 = 6.708
Example 10: Find the smallest number that must be added to 2400 to make it a perfect square.
Solution.
4 2400 48
16
88 800
704
96
\ Number to be added = (49)
2
2400 = 2401 2400 = 1
Cube Root
The cube root of a number is that number the cube of which itself gives the given number ie, the cube root of 64 is 4. The
cube root of a number is denoted by the symbol
3
.
The expression
3
8
is read as cube eight, or the cube root of eight.
To Find the Cube Root of an Integer
(i) By the method of prime factors: When a given number is a perfect cube, we resolve it into prime factors and take
the product of prime factors, choosing one out of every three.
Example 11: Find the cube root of 74088.
Solution.
2 74088
2 37044
2 18522
7 9261
7 1323
7 189
3 27
3 9
3 3
1
74088 = 2
3
7
3
3
3
3
74088
= 2 7 3 = 42
To Find the Cube Root of a Decimal
Example 12: Find the cube root of 19.683.
Numerical Aptitude Number System
For Any Guidance Call Our Expert at +91 8800734161

Click Here For Hard Copy of this Study Materials:
http://bankpoclerk.com/community/study-kit/ibps-po-capsule
Numerical Aptitude Square Root & Cube Root
2013 bankpoclerk.com
Solution.
3
19.683
=
3
19683
1000
=
3
3
19683
1000
=
3 9
3 3
3
10
=
3
3
10
=
27
10
= 2.7
3 19683
3 6561
3 2187
3 729
3 243
3 81
3 27
3 9
3 3
1
19683 = 3
9

1.
144
is equal to
(1) 12 (2) 14
(3) 16 (4) 18
(5) None of these
2. Square root of 2025 is
(1) 35 (2) 45
(3) 55 (4) 65
(5) None of these
3. Find the square root of 9104976
(1) 322 (2) 324
(3) 326 (4) 328
(5) None of these
4. Find the square root of 906.01.
(1) 30.1 (2) 31.1
(3) 30.11 (4) 31.11
(5) None of these
Numerical Aptitude Number System
For Any Guidance Call Our Expert at +91 8800734161

Click Here For Hard Copy of this Study Materials:
http://bankpoclerk.com/community/study-kit/ibps-po-capsule
ANSWERS
1. (1) 2. (2) 3. (2) 4. (1)
EXPLANATIONS
1. 144 = 2 2 2 2 3 3

144
= 2 2 3 = 12
2. 2025 = 5 5 9 9
2025
= 45
3. 3 104976 324
9
62 149
124
644 2576
2576

4.
3 906.01 30.1
9
601 601
601

906.01
= 30.1
Numerical Aptitude Square Root & Cube Root
2013 bankpoclerk.com
In simplification of an expression there are certain laws which should be strictly adhered to. These laws are as follows:
VBODMAS Rule
This rule gives the correct sequence in which the mathematical operation are to be executed so as to find out the value
of a given expression.
Here, V stands for Vinculum (or Bar), B stands for Bracket, O stands for Of, D stands for Division, M
stands for Multiplication, A stands for Addition and S stands for Subtraction.
(1) Here, VBODMAS gives the order of simplification. Thus, the order of performing the mathematical operations in a
given expression are
First : Vinculum or line bracket or bar
Second: Bracket
Third: Of
Fourth: Division
Fifth: Multiplication
Sixth: Addition &
Seventh: Subtraction
The above order should strictly be followed.
(2) There are four types of brackets.
(i) Square brackets [ ]
(ii) Curly brackets { }
(iii) Circular brackets ( )
(iv) Bar or Vinculum
Thus, in simplifying an expression all the brackets must be removed in the order , ( ), { } and [ ].
Modulus of a Real Number
The modulus of a real number x is defined as
= x, if a > 0
|x|
= x, if a < 0
5
Simplification
Numerical Aptitude Simplification
2013 bankpoclerk.com
Basic Formulae
(i) (a + b)
2
= a
2
+ 2ab + b
2
(ii) (a b)
2
= a
2
2ab + b
2
(iii) (a + b)
2
(a b)
2
= 4ab
(iv) (a + b)
2
+ (a b)
2
= 2(a
2
+ b
2
)
(v) (a
2
b
2
) = (a + b) (a b)
(vi) (a + b + c)
2
= a
2
+ b
2
+ c
2
+ 2(ab + bc + ca)
(vii) (a
3
+ b
3
) = (a + b) (a
2
ab + b
2
)
(viii) (a
3
b
3
) = (a b) (a
2
+ ab + b
2
)
(ix) (a
3
+ b
3
+ c
3
3abc) = (a + b + c), (a
2
+ b
2
+ c
2
ab bc ca)
(x) a
3
+ b
3
+ c
3
= 3abc, if a + b + c = 0
Example 1: Simplify 1005 + 500 10 80.
Solution. 1005 + 500 10 80 = 1005 + 490 80 = 1495 80 = 1415
Example 2: If a * b = 2(a + b), then what is the value of 5 * 2?
Solution. 5 * 2 = 2(5 + 2) = 2 7 = 14
Example 3: If x =
1 1
3 3
3 3

+ , then what will be the value of 3x


3
9x?
Solution. 3x
3
9x = 3[3
1/3
+ 3
1/3
]
= 3[3 + 3
1
+ 3(3
1/3
+ 3
1/3
)] 9[3
1/3
+ 3
1/3
]
=
1/3 1/3 1/3 1/3
1
3 3 9(3 3 ) 9(3 3 )
3

_
+ + + +

,
= 9 + 1
= 10
Example 4: 3/5 part of the students in a class are the girls and remaining are the boys. If 2/9 part of the girls and 1/4 part
of the boys are absent, then what part of total students is present.
Solution. Let total number of students = x
Number of girls =
3
5
x
Number of boys =
2
5
x
Number of absent students =
2 3 1 2
of of
9 5 4 5
x x
_ _
+

, ,
=
2 1
15 10
x x +
=
7
30
x
Present students =
7
1
30
x
_


,
=
23
30
x
Example 5: Simplify (2
10
2
9
) (2
8
2
7
).
Solution. (2
10
2
9
) (2
8
2
7
) = 2
9
(2 1).2
7
(2 1) = 2
9+7
= 2
16
Numerical Aptitude Number System
For Any Guidance Call Our Expert at +91 8800734161

Click Here For Hard Copy of this Study Materials:
http://bankpoclerk.com/community/study-kit/ibps-po-capsule
Numerical Aptitude Simplification
2013 bankpoclerk.com
1. What is
1
th
6
of 3?
(1) 6 (2) 3 (3) 1/2 (4) 1/3
(5) None of these
2. What is 10 + 8 equal to?
(1) 2 (2) 18 (3) 10 (4) 8
(5) None of these
3. Multiply 0.932 by 100.
(1) 932 (2) 9.32 (3) 93.2 (4) 9320
(5) None of these
4. Divide 0.045 by 100.
(1) 0.0045 (2) 0.00045
(3) 0.000045 (4) 0.45
(5) None of these

Numerical Aptitude Number System


For Any Guidance Call Our Expert at +91 8800734161

Click Here For Hard Copy of this Study Materials:
http://bankpoclerk.com/community/study-kit/ibps-po-capsule
ANSWERS
1. (3) 2. (2) 3. (3) 4. (2) 5. (3)
EXPLANATIONS
1.
1
th
6
of 3 =
1
3
6

=
1
2
2. 10 + 8 = 18
3. 0.932 100 = 93.2
4.
0.045
100
= 0.00045
Numerical Aptitude Square Root & Cube Root
2013 bankpoclerk.com
Highest Common Factor
The highest common factor of two or more given numbers is the largest of their common factors. It is known as Greatest
Common Divisor also.
eg, Factors of 20 are 1, 2, 4, 5, 10, 20
Factors of 36 are 1, 2, 3, 4, 6, 9, 12, 18, 36
Here greatest and common factor of 20 and 36 is 4.
HCF of 20 and 36 is 4.
Least Common Multiple
The least common multiple of two or more given numbers is the least of their common multiples.
eg, Multiple of 25 are 25, 50, 75, 100, 125, 150, 175, ....
Multiple of 30 are 30, 60, 90, 120, 150, 180, 210, ....
Here 150 is least common multiple of 25 and 30
LCM of 25 and 30 is 150.
Using Division Method
Example 1: Determine the HCF and LCM of 36, 48, 64 and 72.
Solution. To find HCF
36 ) 48 ( 1
36
12 ) 36 ( 3
36

12 ) 64 ( 5
60
4 ) 12 ( 3
12

4 ) 72 (18
4
32
32
HCF of 36, 48, 64 and 72 is 4.
6
HCF & LCM
Numerical Aptitude HCF & LCM
2013 bankpoclerk.com
To find LCM
2 36, 48, 64, 72
2 18, 24, 32, 36
2 9, 12, 16, 18
2 9, 6, 8, 9
3 3, 3, 4, 9
3 3, 1, 4, 3
4 1, 1, 4, 1
1, 1, 1, 1
LCM = 2 2 2 2 3 3 4 = 1728
HCF and LCM of Fractions
HCF of fraction =
HCF of Numerators
LCMof Denominators
LCM of fraction =
LCM of Numerators
HCF of Denominators
Example 2: Determine the HCF and LCM of
14 42 21
, , .
33 55 22
Solution. HCF of
14 42 21
, , .
33 55 22
=
HCF of 14,42,21
LCMof 33,55,22
Now, to determine HCF of 14, 42 and 21
14 ) 42 ( 3
42

14 ) 21 ( 1
14
7 ) 14 ( 2
14

HCF of 14, 42, 21 is 7
And to determine LCM of 33, 55, 22
11 33, 55, 22
3, 5, 2
LCM of 33, 55 and 22 = 11 3 5 2 = 330
Hence, Required HCF =
7
330
Now, LCM of
14 42 21
, ,
33 55 22
=
LCM of 14,42,21
HCF of 33,55,22
Numerical Aptitude Number System
For Any Guidance Call Our Expert at +91 8800734161

Click Here For Hard Copy of this Study Materials:
http://bankpoclerk.com/community/study-kit/ibps-po-capsule
Numerical Aptitude HCF & LCM
To determine LCM of 14, 42, 21.
7 14, 42, 21
2 2, 6, 3
3 1, 3, 3
1, 1, 1
LCMof 14, 42, 21 = 7 2 3 = 42
To determine HCF of 33, 55, 22
33 ) 55 ( 1
33
22 ) 33 ( 1
22
11 ) 22 ( 2
22

11 ) 22 ( 2
22

HCF of 33, 55, 22 = 11
Hence, Required LCM =
42
11
The least number which is exactly divisible by a, b and c is the LCM of a, b, c.
The greatest number that will divide a, b, c is the HCF of a, b, c.
If x is a factor of a and b, then x is also a factor of a + b, a b and ab.
HCF of given numbers must be a factor of their LCM.
The product of the LCM and HCF of two numbers is equal to the product of the two numbers.

1. How many numbers less than 10,000 are there which


are divisible by 21, 35 and 63?
(1) 33 (2) 32 (3) 38 (4) 31
(5) None of these
2. Find the side of the largest possible square slabs which
can be paved on the floor of a room 2m 50 cm long and
1 m 50 cm broad. Also find the number of such slabs to
pave the floor.
(1) 40, 18 (2) 30, 15 (3) 50, 15 (4) 20, 25
(5) None of these
Numerical Aptitude Number System
For Any Guidance Call Our Expert at +91 8800734161

Click Here For Hard Copy of this Study Materials:
http://bankpoclerk.com/community/study-kit/ibps-po-capsule
ANSWERS
1. (4) 2. (3)
EXPLANATIONS
1. LCM (21, 35, 63)= 315
The numbers less than 10,000 which are divisible by
315 are given by
10, 000
,
315
1
1
]
i.e., the integral part when
10,000 is divided by 315 is 31.
The required answer is 31.
2. HCF (250, 150) = 50 cm
The number of slabs =
250 150
50 50

= 15
Numerical Aptitude Square Root & Cube Root
Example 1: Ajay, Akshay and Saroj cover a distance of
1
33 km
2
, 33500 m and 290 hactometre respectively in an hour..
Who has the maximum speed?
Solution. Distance covered by Ajay =
1
33 km
2
= 67 1000 m = 33500 m
Distance covered by Akshay = 33500 m
Distance covered by Saroj = 290 hactometre
= 290 100 m = 29000 m
Since, distance covered by Ajay and Akshay are maximum and equal. Hence, Ajay and Akshay have maximum speed.

1. Ramesh gave milk to have to his three sons Harish,


Shayam and Ajay in three pots of the shape hemisphere,
cube and cuboid. If radius of hemisphere pot is 5 cm,
side of cubic pot is 5 cm and sides of cuboid pot are 5
cm 5 cm 6 cm, then who will get more milk?
(1) Harish (2) Shayam
(3) Ajay (4) Equal to all
(5) None of these
2. The velocity of sound in first medium is 320 m/s and in
second medium is 1152 km/h. In which medium velocity
of sound is maximum?
(1) First (2) Second
(3) Equal in both
(4) Cant be determined
(5) None of these
7
Orders of Magnitude
ANSWERS
1. (1) 2. (3)
EXPLANATIONS
1. Volume of hemispherical pot =
3
2
3
r
=
3
2 22
(5)
3 7

= 261.90 cm
3
Volume of cubic pot = (5)
3
= 125 cm
3
Volume of cuboid pot = 5 5 6 = 150 cm
3
Since, volume of hemispherical pot is maximum.
Hence, Harish will get the maximum milk.
2. Velocity of sound
In first medium = 320 m/s
In second medium = 1152 km/h
=
5
1152 m/s
18

= 320 m/s
Hence, velocity of sound is same in both mediums.
2013 bankpoclerk.com
Numerical Aptitude Square Root & Cube Root
2013 bankpoclerk.com
Unitary method is the basic area of arithmetic.
Direct Proportion
Two quantities are said to be directly proportional, if on the increase in one the other increases proportionally or on the
decrease in one the other decreases proportionally.
eg, More the numbers of articles, more is the cost.
More the number of workers, more is the work done.
Less the number of articles, less is the cost.
Less the number of workers, less is the work done.
Indirect Proportion
Two quantities are said to be indirectly proportional, if on the increase in one the other decreases proportionally or on the
decrease in one the other increases proportionally.
eg, More the number of workers, less is the number of days required to finish a work. More the speed, less is the time
taken to cover a certain distance.
Less the number of workers, more is the number of days required to finish a work. Less the speed, more is the time taken
to cover a certain distance.
Chain Rule
When a series of variables are connected with one another, that we know how much of the first kind is equivalent to a
given quantity of second, how much of the second is equivalent to a given quantity of the third and so on. The rule by which
we can find how much of the last kind is equivalent to a given quantity of the first kind is called the Chain Rule.
Example 1: If 12 apples cost 216, what is the cost of 3 dozen apples ?
Solution. Let the required cost be x. Also, 3 dozen apples = 35 apples more apples, more cost (Direct Proportion);
Apples Cost in
12 216
36 x
12 x = 36 216 x =
36 216
12
= 648
8
Unitary Method
Numerical Aptitude Unitary Method
2013 bankpoclerk.com
1. Cost of 24 pens is 96. Find the cost of 16 such pens.
(1) 66 (2) 64
(3) 62 (4) 68
(5) None of these
2. A bus travels 240 km in 3 h. How long will it take to
travel 360 km?
(1) 5 h. (2) 4 h
(3)
1
4
2
h (4)
1
5
2
h
(5) None of these
3. The temperature dropped 18C in the last 24 days. If
the rate of temperature drop remains constant, then
how many degrees will the temperature drop in the
next 32 days?
(1)
1
24 C
2
(2) 22 C
(3)
1
22 C
2
(4) 24 C
(5) None of these

Numerical Aptitude Number System


For Any Guidance Call Our Expert at +91 8800734161

Click Here For Hard Copy of this Study Materials:
http://bankpoclerk.com/community/study-kit/ibps-po-capsule
ANSWERS
1. (2) 2. (3) 3. (4)
EXPLANAIONS
1. Let the required cost be Rs. x. Then,
Less pens, less cost (Direct Proportion)
Pens Cost in Rs.
24 96
16 x
x =
16
96
24

= Rs. 64
2. Let the required number of hours be x.
More distance, More hours (Direct Proportion)
Distance (in km) Hours
240 3
360 x
x =
360
3
240

=
9
2
=
1
4 h
2
3. Let the required drop in temperature be x.
More days, More drop in temperature (Direct
Proportion)
Days Drop in temperature
24 18
32 x
x =
32
18
24

= 24 C
Numerical Aptitude Square Root & Cube Root
2013 bankpoclerk.com
1. Six years ago, the ratio of the ages of Kunal and Sagar
was 6 : 5. Four years later, the ratio of their ages will be
11 : 10. What is Sagars age at present?
(1) 16 years (2) 19 years
(3) 22 years (4) 25 years
(5) None of these
2. The total of the ages of Jayant, Prem and Saransh is 93
years. Ten years ago, the ratio of their ages was 2 : 3 :
4. What is the present age of Saransh?
(1) 44 years (2) 36 years
(3) 33 years (4) 38 years
(5) None of these
3. The ratio of the present ages of two brothers is 1 : 2 and
5 years back, the ratio was 1 : 3. What will be the ratio
of their ages after 5 years?
(1) 3 : 4 (2) 3 : 2
(3) 3 : 5 (4) 5 : 6
(5) None of these
9
Problem on Ages
ANSWERS
1. (1) 2. (4) 3. (3)
EXPLANATIONS
1. Let the ages of Kunal and Sagar 6 years ago be 6x and
5x years respectively.
Then,
(6 6) 4
(5 6) 4
+ +
+ +
x
x
=
11
10
= 10(6x + 10) = 11(5x + 10)
= 5x = 10 x = 2
Sagars present age = (5x + 6) = 16 years.
2. Let the ages of Jayant, Prem and Saransh 10 years ago
be 2x, 3x and 4x years respectively.
Then, (2x + 10) + (3x + 10) + (4x + 10) = 93 9x = 63
x = 7.
Saranshs present age
= (4x + 10)
= 38 years.
3. Let the present ages of the two brothers be x years and
2x years respectively.
Then,
5 1
2 5 3
x
x

Numerical Aptitude Square Root & Cube Root


2013 bankpoclerk.com

The average of a given number of quantities of the same kind is expressed as


Average =
Sum of the quantities
Number of the quantities
Average is also called the Arithmetic Mean.
Also, Sum of the quantities = Average Number of the quantities
Number of quantities =
Sum of the quantities
Average
If all the given quantities have the same value, then the number itself is the average.
If all the given quantities are not all the same, then the average of the given quantities is
always greater, than the smallest number and always less than the largest number. Equivalently,
at least one of the numbers is less than the average and atleast one is greater then the average.
If each of the given quantities is increased by a constant p, than their average is also increased
by p.
If each of the given quantities is decreased by a constant p, then their average is also decreased
by p.
If each of the given quantities is multiplied by a constant p, then their average is also multiplied
by p.
Whenever the given quantities form an arithmetic sequence and if the given quantities has
odd terms, then the average is the middle term in the sequence and if the given quantities has
even terms, then the average of the sequence is the average of the middle two terms.
In order to calculate the weighted average of a set of numbers, multiply each number in the set
by the number of times it appears, add all the products and divide by the total number of
numbers in the set.
If the speed of an object from A to B is x km/h and from B to A is y km/h,then the average speed
during the whole journey is
2
km/h
xy
x y +
.
10
Average
Numerical Aptitude Average
2013 bankpoclerk.com
If the average of N
1
quantities is x and N
2
quantities is y then the average of total (N
1
+ N
2
)
quantities is given by
1 2
1 2
( ) N x N y
N N
+
+
Example 1: What is the average of first five even numbers
Sol uti on. The first five numbers are 2, 4, 6, 8, 10
Average =
2 4 6 8 10
5
+ + + +
=
30
5
= 6
Example 2: The average of five consecutive even numbers is 50. What is the largest of these numbers?
Sol uti on. Let the numbers be x 4, x 2, x, x + 2, x + 4.
Average =
Sum of the quantities
Number of the quantities
=
4 2 2 4
5
x x x x x + + + + + +
= 50

5
5
x
= 50
x = 50
So, the numbers are 46, 48, 50, 52, 54.
The largest of these numbers is 54.
Example 3: Average weight of 32 students of a class is 30.5 kg. If weight of a teacher is also included
then average weight is increased by 500 g. What is the weight of the teacher?
Sol uti on. Total weight of 32 students = 30.5 32 = 976 kg
Average weight of (32 students + 1 teacher) = (30.5 + 0.5) = 31 kg
Total weight of (32 students + 1 teacher) = 31 33 = 1023 kg
Weight of teacher = (1023 976) kg = 47 kg
Example 4: The average salary per head of all the employees of an institution is 60. The average
salary per head of 12 officers is 400 and average salary per head of the rest is 56. Find the total
number of employees in the institution.
Sol uti on. Let the total number of employees be x.
Then, 60 =
Total salary of all employees
x
60 =
12 400 ( 12) 56 x
x
+
60x = 12 400 + (x 12) 56 = 4800 + 56x 672
60x 56x = 4800 672
4x = 4128 x = 1032
Hence, the total number of employees is 1032.
Numerical Aptitude Number System
For Any Guidance Call Our Expert at +91 8800734161

Click Here For Hard Copy of this Study Materials:
http://bankpoclerk.com/community/study-kit/ibps-po-capsule
Numerical Aptitude Average
2013 bankpoclerk.com
1. The average weight of 8 persons increases by
2.5 kg when a new person comes in place of
one of them weighing 65 kg. What might be
the weight of the new person?
(1) 82 kg (2) 85 kg
(3) 76.5 kg (4) 80 kg
(5) None of these
2. Sumitra has an average of 56% on her first 7
examinations. How much she should make on
her eighth examination to obtain an average
of 60% on 8 examinations?
(1) 88% (2) 78%
(3) 92% (4) 68%
(5) None of these
3. Ages of A and B are in the ratio of 2 : 3
respectively. Six years later the ratio of their
ages will become 8 : 11 respectively. What is
Bs present age?
(1) 18 years (2) 28 years
(3) 27 years (4) 25 years
(5) None of these
4. The total age of A and B is 12 years more than
that of total age of B and C. C is how many
years younger than A?
(1) C is elder than A (2) 26
(3) 12 (4) 25
(5) None of these

Numerical Aptitude Number System


For Any Guidance Call Our Expert at +91 8800734161

Click Here For Hard Copy of this Study Materials:
http://bankpoclerk.com/community/study-kit/ibps-po-capsule
ANSWERS
1. (2) 2. (1) 3. (3) 4. (3)
EXPLANATIONS
1. By short cut: The weight of the new person
= 8 2.5 + 65 = 85 kg
2. By given condition,
56 7 + x = 60 8
x = 480 392 = 88%
4. By given condition, A + B
= 12 + B + C A C = 12
C is 12 years younger than A.
Numerical Aptitude Percentage
2013 bankpoclerk.com

Per cent means per hundred. It is denoted by % (symbol). Here x% means x per hundred or
.
100
x
Thus, any percentage can be converted into an equivalent fraction by dividing it by 100.
eg 20% =
20 1
;
100 5

150% =
150 3
100 2

Also, any fraction or decimal can be converted into its equivalent percentage by multiplying with
100.
eg
1 1
100
5 5

= 20%;
3 3
100
2 2

= 150%.

1. Percentage increase =
Increase
100
Original value

2. Percentage decrease =
Decrease
100
Original value

3. If the price of the commodity increases by r% then the reduction in consumption so as not to
increase the expenditure is
100 %
100
r
r
1

1
+
]
4. If the price of the commodity decreases by r% then the reduction in consumption so as not to
increase the expenditure is
100 %
100
r
r
1

]
5. If As income is r% more than Bs income then Bs income is less than As income by
100 %
100
r
r
1

1
+
]
.
11
Percentage
Numerical Aptitude Percentage
2013 bankpoclerk.com
6. If As income is r% less than Bs income then Bs income is more than As income by
100 %
100
r
r
1

]
.
7. Let the population of a town be P and it increases at the rate of r% per annum, then
(1) Population after n years =
1
100
n
r
P
_
+

,
(2) Population n years ago =
1
100
n
P
r _
+

,
8. Let the present value of the machine be P and if it depreciates at the rate of r% per annum.
(1) Value of machine after n years =
1
100
n
r
P
_


,
(2) Value of machine n years ago =
_


,
1
100
n
p
r
Example 1: Express 3/2 as rate per cent.
Sol uti on.
3
2
=
3
100 %
2
_


,
= 150%
Example 2: Find 25% of 1000.
Sol uti on. 25% of 1000 =
25
1000
100

= 250
Example 3. What per cent of 6 is 144?
Sol uti on. Required percentage =
144
100 %
6
_


,
= 2400%
Example 4: What per cent of 2.5 kg is 15 g?
Sol uti on. Required percentage =
15
100 %
2.5 1000
_

,
= 0.6%
Example 5. If the price of tea falls by 12%, by how much per cent must a housewife increase its
consumption, so as not to decrease its expenditure on tea?
Sol uti on. (Short cut method)
Increase % in consumption =
100 %
100
r
r

' ;


=
12
100 %
100 12

' ;


=
12
100 %
88
_


,
=
150
%
11
=
7
13 %
11
Numerical Aptitude Number System
For Any Guidance Call Our Expert at +91 8800734161

Click Here For Hard Copy of this Study Materials:
http://bankpoclerk.com/community/study-kit/ibps-po-capsule
Numerical Aptitude Percentage
2013 bankpoclerk.com

1. The difference of two numbers is 20% of the


larger number. If the smaller number is 20,
then the larger number is:
(1) 25 (2) 46
(3) 27 (4) 82
(5) None of these
2. When any number is divided by 12, then
dividend becomes 1/4th of the other number.
By how much percent first number is greater
than the second number?
(1) 165 (2) 200
(3) 300 (4) 400
(5) None of these
Numerical Aptitude Number System
For Any Guidance Call Our Expert at +91 8800734161

Click Here For Hard Copy of this Study Materials:
http://bankpoclerk.com/community/study-kit/ibps-po-capsule
ANSWERS
1. (1) 2. (2)
EXPLANATIONS
1. Let the larger number be x.
Then, x 20 =
20 1
20
100 5
x x x

4
5
x
= 20 x
=
5
20
4

' ;

= 25
2. Let the numbers be x and y. Then,
3 .
12 4
x y
x y
Required percentage =
100 %
x y
y
_


,
=
2
100 %
y
y
_


,
= 200%
Numerical Aptitude Percentage
2013 bankpoclerk.com
-
The price at which an article is purchased, is called the cost price or CP.

The price at which an article is sold, is called the selling price or SP.

Gain or Profit = SP CP
Gain per cent or Profit per cent =
Gain Profit
100 or 100
CP CP
_ _


, ,
SP =
100 Profit %
CP
100
+ _


,
Similarly, Loss = CP SP
Loss per cent =
Loss
100
CP
_


,
; SP =
(100 Loss%)
CP
100

The Profit and Loss per cent is always calculated on the cost price.
If a trader professes to sell his goods at CP but uses false weight, then Gain per cent or Profit per
cent
=
Error
100 %
True Value Error
_

,
-
Price that is indicated or marked on the article is called marked price or MP.
-
It is reduction given on the Marked Price or List Price of an article.
d per cent =
100 discount
;
MP

Selling Price =
(100 %)
100
d
MP

If a trader gets x% profit and x% loss in selling two different articles, then in over all transaction,
there is always a loss which is given by
12
Profit and Loss
Numerical Aptitude Profit and Loss
2013 bankpoclerk.com
Loss % =
2
10
x _

,
Example 1: A chair is bought for 1950 and sold at 2340. Find the gain per cent.
Sol uti on. CP = 1950 and SP = 2340
Gain = (2340 1950) = 390
Gain % =
390
100 %
1950
_


,
= 20%
Example 2: A radio is bought for 780 and sold at 650. Find the loss per cent.
Sol uti on. CP = 780 and SP = 650
Loss = CP SP = (780 650) = 130
Loss % =
_


,
130
100
780
=
2
16 %
3
Example 3: A book is bought for 80 and sold at the gain of 5%. Find the selling price.
Sol uti on. CP = 80, Gain = 5%
SP = 105% of 80 =
105
80
100
_


,
= 84
Example 4: If cost price of 15 articles is equal to the selling price of 12 articles, then find the gain
per cent.
Sol uti on. Let cost price of each article = 1
Then, Cost price of 15 articles = 15
Selling price of 12 articles = 15
But Cost price of 12 articles = 12
Profit = (15 12) = 3
Profit % =
3
100
12

= 25%
Example 5: What is the equivalent discount of three consecutive discount 30%, 20% and 5%?
Sol uti on. Let MP = 100
SP = 95% of 80% of 70% of 100 =
95 80 70
100
100 100 100

= 53.20
Required equivalent discount = (100 53.20) = 46.80
Example 6: By selling 66 m of cloth a person gains the cost price of 22 m. Find the gain per cent.
Sol uti on. Let CP of 1 m cloth = 1
Then, Gain % =
gain
100
CP
=
CP of 22 m cloth
100
CP of 66 m cloth
=
22
100
66
=
1
33 %
3
Numerical Aptitude Number System
For Any Guidance Call Our Expert at +91 8800734161

Click Here For Hard Copy of this Study Materials:
http://bankpoclerk.com/community/study-kit/ibps-po-capsule
Numerical Aptitude Profit and Loss
2013 bankpoclerk.com
1. A man purchased a box full of pencils at the
rate of 7 for 9 and sold all of them at the
rate of 8 for 11. In this transaction, he gained
10. How many pencils did the box contain?
(1) 111 (2) 112
(3) 114 (4) 116
(5) None of these
2. A man bought a number of clips at 3 for a rupee
and an equal number at 2 for a rupee. At what
price per dozen should he sell them to make
a profit of 20%?
(1) 9 (2) 10
(3) 6 (4) 7
(5) None of these
3. A man buys eggs at 2 for Re. 1 and an equal
number at 3 for 2 and sells the whole at 5
for 3. His gain or loss percent is:
(1)
2
2
7
% loss (2)
6
3
7
% gain
(3)
2
3
7
% loss (4)
6
2
7
% gain
(5) None of these
4. A man bought some oranges at 10 per dozen
and bought the same number of oranges at
8 per dozen. He sold these oranges at 11
per dozen and gained 120. The total number
of oranges bought by him was:
(1) 55 dozens (2) 80 dozens
(3) 90 dozens (4) 60 dozens
(5) None of these

Numerical Aptitude Number System


For Any Guidance Call Our Expert at +91 8800734161

Click Here For Hard Copy of this Study Materials:
http://bankpoclerk.com/community/study-kit/ibps-po-capsule
ANSWERS
1. (2) 2. (3) 3. (4) 4. (4) 5. (3)
EXPLANATIONS
1. Suppose, number of pencils bought
= L.C.M. of 7 and 8 = 56.
C.P. of 56 pencils = -
9
56
7
_


,
= - 72.
S.P. of 56 pencils = -
11
56
8
_


,
= - 77.
Now, - 5 are gained on 56 pencils.
So, - 10 are gained on
56
10
5
_


,
= 112 pencils.
2. Suppose he bought 1 dozen clips of each kind.
C.P. of 2 dozens
= -
1 1
12 12
3 2
_
+

,
= - 10
S.P. of 2 dozen = 120% of - 10
= -
120
10
100
_


,
= - 12
Hence, S.P. per dozen = - 6.
3. Suppose he buy 6 eggs of each kind.
C.P. of 12 eggs = -
1 2
6 6
2 3
_
+

,
= - 7.
S.P. of 12 eggs = -
3
12
5
_


,
= - 7.20.
Gain=
0.20
100 %
7
_


,
=
6
2 %.
7
4. C.P. of 2 dozen oranges = - (10 + 8) = -
18.
S.P. of 2 dozen oranges = - 22.
If profit is - 4, orange bought = 2 dozen.
If profit is - 120, oranges bought
=
2
120
4
_


,
dozens = 60 dozens.
Numerical Aptitude Percentage
2013 bankpoclerk.com

The ratio of two quantities a and b is the fraction


a
b
and is expressed as a : b. Here a is the first term
or antecedent and b is the second term or consequent. Since the ratio expresses the number of times one
quantity contains the other, it is an abstract (without units) quantity.
A ratio remains unaltered if its numerator and denominator are multiplied or divided by the same
number. eg, 4 : 3 is the same as (4 10) : (3 10) ie, 40 : 30.
20 : 15 is the same as
20 15
:
5 5
_ _

, ,
ie, 4 : 3.
A ratio is said to be a ratio of greater or less inequality or of equality according as antecedent is
greater than, less than or equal to consequent.
If a > b, then a : b is called a ratio of greater inequality (eg, 4 : 3, 5 : 2, 11 : 3, ...)
If a < b, then a : b is called a ratio of less inequality (eg, 3 : 4, 2 : 5, 3 : 11, ...)
If a = b, then a : b is called a ratio of equality (eg, 1 : 1, 3 : 3, 5 : 5, ...)
From this we find that
(i) If a > b and some positive number is added to each term of a : b, then the ratio is diminished.
If a > b, then (a + x) : (b + x) < a: b.
(ii) If a < b and some positive number is added to each term of a : b, then the ratio is increased.
If a < b, then (a + x) : (b + x) < a : b.
(iii) If a = b and some positive number is added to each term of a : b, then the ratio is unaltered.
If a = b, then (a + x) : (b + x) = a : b
- -
Duplicate Ratio: a
2
: b
2
is called duplicate ratio of a : b.
Triplicate Ratio: a
3
: b
3
is called triplicate ratio of a : b.
Sub-Duplicate Ratio: : a b is called sub-duplicate ratio of a : b.
Sub-triplicate Ratio :
3 3
: a b is called sub-triplicate ratio of a : b.
13
Ratio & Proportion
Numerical Aptitude Ratio & Proportion
2013 bankpoclerk.com
Compound Ratio : ab : cd is the compound ratio of a : c and b : d. It is the ratio of the product of the
antecedents to that of the consequents of two or more given ratios.
Inverse Ratio :
1 1
:
a b
is the inverse ratio of a : b.
Componendo and Divedendo: If
,
a c
b d

then
a b c d
a b c d
+ +

When two ratios are equal, they make a proportion. ie, if


,
a c
b d

then a, b, c and d are in proportion.


This is represented as a : b : : c : d and is read as a is to b as c is to d.
When a, b, c and d are in proportion, then a and d are called the Extremes and b and c are called the
Means. also, Product of the Means = Product of the Extremes ie, bc = ad.

If three quantities a, b and c are such that a : b : : b : c, then b
2
= ac and a, b and c are in continued
proportions. Also, the quantity c is called the third proportion of a and b.

If four quantities a, b, c and x are such that a : b : : b : c, then ax = bb and x is called the fourth
proportion of a, b, and c.

If three quantities a, b and x are such that a : x : : x : b, then x
2
= ab and x is called the mean of a and
b. Also, If a : b = c : d, then the following properties hold good.
(i) b : a = d : c (Invertendo)
(ii) a : c = b : d (Alter nendo)
(iii) (a + b) : b = (c + d) : d (Componendo)
(iv) (a b) : b = (c d) : d (Dividendo)
(v)
a b c d
a b c d
+ +


(Componendo - Dividendo)

If two quantities x and y are related in such a way that as the quantity x changes it also brings a
change in the second quantity y, then the two quantities are in variation.

The quantity x is in direct variation to y if an increase in x makes y to increase proportionally. Also
a decrease in x makes y to decrease proportionally it can be expressed as x = ky, where, k is called the
constant of proportionality. eg, Cost is directly proportional to the number of articles bought.
-
The quantity x is in inverse variation to y if an increase in x makes y to decrease proportionally.
Also, a decrease in x makes y to increase proportionally. It can be expressed as
.
k
x
y

where, k is called
Numerical Aptitude Number System
For Any Guidance Call Our Expert at +91 8800734161

Click Here For Hard Copy of this Study Materials:
http://bankpoclerk.com/community/study-kit/ibps-po-capsule
Numerical Aptitude Ratio & Proportion
2013 bankpoclerk.com
the constant of proportionality. eg, The time taken by a vehicle in covering a certain distance is inversely
proportional to the speed of the vehicle.

If there are more than two quantities x, y and z and x varies with both y and z, then x is in joint
variation to y and z. It can be expressed as x = kyz, where, k is the constant of proportionality. eg, Men
doing a work in some number of days working certain hours a day.
-
If an amount A is distributed in the ratio a : b, then
First part =
a
A
a b

+
Second part =
b
A
a b

+
Example 1: Divide 60 in the ratio of 1 : 3
Sol uti on. We have 1 + 3 = 4
First part =
1
60
4

= 15
Second part =
3
60
4

= 45
Thus, the required parts are 15 and 45.
Example 2: The ratio of boys to girls in a science class of 28 is 16 : 12. Express it in simplest ratio.
Sol uti on. 16 : 12 = 8 : 6 = 4 : 3
Which is the ratio in its simplest form.
Example 3: Compare the ratio of the third proportion of 6 and 5 with the fourth proportion of 4, 3
and 10.
Sol uti on. Third proportion of 6 and 5 is 6 : 5 : : 5 : x
x =
25
6
Fourth proportion of 4, 3 and 10 is 4 : 3 : : 10 : y
y =
3 10
4

=
15
2
Required ratio =
25 15
:
6 2
= 5 : 9
Numerical Aptitude Number System
For Any Guidance Call Our Expert at +91 8800734161

Click Here For Hard Copy of this Study Materials:
http://bankpoclerk.com/community/study-kit/ibps-po-capsule
Numerical Aptitude Ratio & Proportion
2013 bankpoclerk.com
1. Ratio of the earnings of A and B is 4 : 7. If the
earnings of A increase by 50% and those of B
decrease by 25%, the new ratio of their
earnings becomes 8 : 7. What are As earnings?
(1) 25,000 (2) 26,000
(3) 29,000 (4) Data Inadequate
(5) None of these
2. What least number must be subtracted from
each of the numbers 14, 17, 34 and 42 so that
the remainders may be proportional?
(1) 4 (2) 3
(3) 2 (4) 9
(5) None of these
3. In a mixture of 60 litres, the ratio of milk and
water is 2 : 1. If this ratio is to be 1 : 2, then
the quantity of water to be further added is:
(1) 30 litres (2) 70 litres
(3) 40 litres (4) 60 litres
(5) None of these
4. The fourth proportional to 5, 8, 15 is :
(1) 22 (2) 24
(3) 23 (4) 20
(5) None of these

Numerical Aptitude Number System


For Any Guidance Call Our Expert at +91 8800734161

Click Here For Hard Copy of this Study Materials:
http://bankpoclerk.com/community/study-kit/ibps-po-capsule
ANSWERS
1. (4) 2. (3) 3. (1) 4. (2)
EXPLANATIONS
1. Let the original earnings of A and B be 4x
and 7x.
New earnings of A = 150% of 4x
=
150
4
100
x
_


,
=
600
100
x
= 6x.
New earnings of B = 75% of 7x
Numerical Aptitude Percentage
2013 bankpoclerk.com
-
When two or more than two persons run a business jointly, they are called partners in the business
and the deal between them is known as partnership.
Partnership is of two types
1. Simple Partnership
2. Compound Partnership
1. Simple Partnership: When investments of all the partners are for the same period of time, the
profit or loss is distributed among the partners in the ratio of their original investments.
Suppose A and B invest p and q respectively for a year in a business, then at the end of the year.
Share of As profit (loss) : Share of Bs profit (loss) = p : q.
2. Compound Partnership: When investments of all the partners are for different period of time,
then equivalent capitals are calculated for a unit of time and the profit or loss is divided in the ratio of
the product of time and investment.
Suppose A and B invest p and q for x months and y months respectively, then Share of As profit
(loss): Share of Bs profit (loss) = px : qy.
Partners are of two types
(i) Working Partner, and
(ii) Sleeping Partner
(i) Working Partner: A partner who manages the business is called a working partner.
(ii) Sleeping Partner: A partner who only invests the money is called a sleeping partner.
Example 1: A and B started a business with capitals of 25000 and 40000 respectively. Find the
share of A and B out of an annual profit of 6500.
Solution. Ratio of shares of A and B = Ratio of their investments = 25000 : 40000 = 5 : 8
As share =
5
6500
13
_


,
= 2500
and Bs share =
8
6500
13

= 4000
14
Partnership
Numerical Aptitude Partnership
2013 bankpoclerk.com
1. A, B and C started a business by investing
28000, 35000 and 14000 respectively. At
the end of a year they got a total profit of
5225. Find As share.
(1) 1740 (2) 1850
(3) 1900 (4) 1650
(5) None of these
2. A, B and C started a business by investing
45000, 55000 and 60000 respectively. At
the end of a year they got a total profit of
11200. Find how much B gets more than A
in the profit.
(1) 700 (2) 750
(3) 710 (4) 780
(5) None of these
Example 2: A, B and C start a business each investing 16000. After 3 months A withdrew 2000,
B withdrew 4000 and C invests 8000 more. At the end of year a total profit of 41580 made. Find the
share of A, B and C
Sol uti on. Ratio of capitals of A, B and C
= (16000 3 + 14000 9) : (16000 3 + 12000 9) : (16000 3 + 24000
9)
= 174000 : 156000 : 264000 = 29 : 26 : 44
As share =
29
41580
99
_


,
= 12180
Bs share =
26
41580
99

= 10920
Cs share =
44
41580
99

= 18480

Numerical Aptitude Number System


For Any Guidance Call Our Expert at +91 8800734161

Click Here For Hard Copy of this Study Materials:
http://bankpoclerk.com/community/study-kit/ibps-po-capsule
ANSWERS
1. (3) 2. (1)
EXPLANATIONS
1. Ratio of shares of A, B and C = Ratio of their
investments = 28000 : 35000 : 14000 = 4 : 5 : 2
As share =
4
5225
11
_


,
= 1900
2. Ratio of shares of A, B and C = Ratio of their
investments = 45000 : 55000 : 60000 = 9:11:12
As share =
9
11200
32
_


,
= 3150
Bs share =
11
11200
32
_


,
= 3850
Bs share more than A = (3850 3150)
= 700
Numerical Aptitude Partnership
2013 bankpoclerk.com
Introduction
It is the rule that is used to determine the mean value of the mixture when the prices of the individual
items being mixed together and the proportion in which they are being mixed are given. Here, the value
of the mixture is always higher than the lowest value and lower than the higher value of the items being
mixed.
According to the Rule of Alligation
Quantity of cheaper
Quantity of dearer
=
Price of dearer Mean price
Mean price Price of cheaper

It can be also expressed as,


Cost price of 1 unit
quantity of cheaper (x)
Mean price (m)
Cost price of 1 unit
quantity of cheaper (x)
(y m) (m x)
(Cheaper quantity : Dearer quantity) = (y m) : (m x)
Where, mean price (m) is the cost price of a unit quantity of the mixture.
Also, if a container contains x units of liquid from which y units are taken out and replaced by water.
After n operations, the quantity of pure liquid is
1
n
y
x
x
1
_
1

1 ,
]
unit.
Example 1: How many kilograms of rice costing 18 per kg must be mixed with 30 kg of rice costing
14 per kg, so that the resultant mixture cost 15 per kg.
Sol uti on. Applying the rule of alligation, we have
( 14) ` ( 18) `
18 - 15 = 3 15 - 14 = 1
Mean price
( 15) `
15
Alligation or Mixture
Numerical Aptitude Alligation or Mixture
2013 bankpoclerk.com

Quantity of cheaper rice


Quantity of dearer rice
=
3
1
If cheaper rice is 3 kg, dearer rice is 1 kg.
If cheaper rice is 30 kg, dearer rice =
30 1
3
_

,
kg = 10 kg
Example 2: In what proportion must a person mix rice 12.00 per kg and 14.40 per kg so as to
make a mixture worth 12.60 per kg?
Sol uti on.
1200 paise CP of 1 kg dearer rice
1440 paise
180 60
Mean price
1260 paise
By the alligation rule,
Quantity of cheaper rice
Quantity of dearer rice
=
180
60
=
3
1
He must mix rice in the ratio 3 : 1.
Example 3: In what proportion must water be mixed with milk to marke gain of 20% by selling it at
cost price?
Sol uti on. Let CP of milk = Re. 1 per litre
SP of 1 litre of mixture = Re. 1,
Profit = 20%
CP of 1 litre mixture =
1
100
120

= Rs.
5
6
Required ratio =
1 5
:
6 6
= 1 : 5
Example 4: Two vessels contain mixture of milk and water in the ratio of 3 : 5 in the first vessel and
in the ratio of 2 : 7 in the second. In what ratio should the contents of these two vessels be mixed so that
the resultant mixture has milk and water in the ratio 1 : 3?
Sol uti on. Here, we can apply the alligation rule taking the concentration of the mixtures. The
concentration of milk in the first vessel is 3/8 and that in the second
2
.
9
Concentration of milk in
first vessel
Mean concentration of milk
in the new mixture
3
8
Concentration of milk in
second vessel
2
8
1
4
3 1 1
8 4 8

1 2 1
4 9 36

The ratio in which the two mixture should be mixed is
1 1
:
36 8
= 8 : 36 = 2 : 9
CP of 1 L of water CO of 1 L of milk
Mean price
` 0 ` 1
`
5
8
1
6
5
6
Numerical Aptitude Number System
For Any Guidance Call Our Expert at +91 8800734161

Click Here For Hard Copy of this Study Materials:
http://bankpoclerk.com/community/study-kit/ibps-po-capsule
Numerical Aptitude Alligation or Mixture
2013 bankpoclerk.com
1. In what ratio, tea at 90 per kg should be
mixed with another tea at 120 per kg to get
a tea of 100 per kg
(1) 2 : 1 (2) 3 : 1
(3) 3 : 2 (4) 4 : 3
(5) None of these
2. Two varieties of rice at 10 per kg and 12
per kg are mixed together in the ratio 1 : 2.
Find the average price of the resulting
mixture
(1) 11 per kg (2) 11.22 per kg
(3) 11.33 per kg (4) 11.44 per kg
(5) None of these
3. On combining two groups of students having
30 and 40 marks respectively in an exam, the
resultant group has an average score of 34.
Find the ratio of the number of students in
the first group to the number of students in
the second group.
(1) 2 : 1 (2) 3 : 2
(3) 3 : 1 (4) 4 : 3
(5) None of these
4. On mixing two classes of students having
average marks 25 and 40 respectively, the
overall average obtained is 30 marks. Find
the number of students in first class if the
second class has 30 students.
(1) 45 (2) 60
(3) 70 (4) 80
(5) None of these

Numerical Aptitude Number System


For Any Guidance Call Our Expert at +91 8800734161

Click Here For Hard Copy of this Study Materials:
http://bankpoclerk.com/community/study-kit/ibps-po-capsule
ANSWERS
1. (1) 2. (3) 3. (2) 4. (2)
EXPLANATIONS
1. CP of 1 kg cheaper tea CP of a kg dearer tea
Mean price
20
` 120 ` 90
10
` 100
Required ratio = 20 : 10 = 2 : 1
2. CP of 1 kg cheaper rice CP of 1 kg dearer
price
Mean price
12 - x
` 12 ` 10
x - 10
` 100
Quantity of cheaper rice
Quantity of dearer rice
=
12
10
x
x

1
2
=
12
10
x
x

x = 11.33 per kg
3. Required ratio =
40 34
34 30

=
6 3
4 2

4.
Students in first class
Students in second class
=
40 30 10 2
30 25 5 1

Numerical Aptitude Partnership


2013 bankpoclerk.com
Work
Work to be done is generally considered as one unit. It may be digging a trench constructing or
painting a wall, filling up or emptying a tank, reservoir or a cistern.
General rules to be followed in the problems on Time and Work
1. If A can do a piece of work in n days, then work done by A in 1 day is I/n. ie,if a person can do some
work in 12 days, he does 1/ 12th of the work in one day.
2. If As 1 days work = 1/n, then A can finish the whole work in n days. ie, if a persons one day work
is 1/10, then he can finish the whole work in 10 days.
3. If A is thrice as good a workman as B, then ratio of work done by A and B = 3 : 1. ie, if a man works
three times as fast as a woman does, then when the work is complete, 3 parts of the work has been
done by the man and 1 part by the woman.
4. If A is thrice as good a workman as B, then ratio of time taken by A and B = 1 : 3. ie, if the woman
takes 15 days to complete the work, then the man takes 5 days to complete the same work.
5. If two persons A and B can individually do some work in a and b days respectively, then A and B
together can complete the same work in ab (a + b) days.
6. The fundamental rules on variation also apply in Time and Work.
(i) Work and men are directly proportional to each other ie, if the work increases, the no. of men
required to do it, also increases, if the work is to be completed in the same number of days.
(ii) Men and days are inversely proportional, ie, if the number of men increases, the number of
days required to complete the same work decreases and vice versa.
(iii) Work and days are directly proportional, ie, if the work increases, the number of days required
also increases, if the work is to be completed with the same number of men and vice versa.
Example 1: Ravi can do a job 10 days. Determine his one day job.
Sol uti on. Ravis 10 days work = 1
Ravis 1 day work =
1
10
Example 2: Tuktuki and Rasmani can do a job alone in 20 days and 30 days respectively. In how
16
Time and Work
Numerical Aptitude Time and Work
2013 bankpoclerk.com
many days the job will be finished, if they work together.
Sol uti on. Tuktukis 1 day work =
1
20
Rasmanis 1 day work =
1
30
(Tuktuki + Rasmani) 1 day work = +
1 1 5 1
20 30 60 12
(Tuktuki + Rasmani) will complete the job in 12 days.
Alternate: Required no. of days =
20 30
20 30

+
=12
Example 3 Mary and Maurice can do a piece of work in 10 days and 15 days respectively. They work
together for 3 days and then Maurice leaves. Mary finishes the remaining work alone. In how many days
is the total work finished?
Sol uti on. (Mary + Maurice)s 1 day work =
1 1 5 1
10 15 30 6
+
(Mary + Maurice)s 3 day work =
3 1
6 2

Remaining work =
1 1
1
2 2

Mary does this piece of work in = 10 days
Mary does the 1/2 piece of work in =
1
10
2
= 5 days
Hence, total number of days to finish the work = 3 + 5 = 8 days
Example 4: Ravi and Rishi can do a piece of work in 24 days, Rishi and Ronit in 30 days, Ronit and
Ravi in 40 days. In how many days will they finish it together and separately?
Sol uti on. (Ravi and Rishi)s 1 days work =1/24
(Rishi and Ronit)s 1 days work = 1/30
(Ronit and Ravi)s 1 days work = 1/40
On adding, we get
2 (Ravi + Rishi + Ronit)s 1 days work = (1/24 + 1/30 + 1/40) = 1/10
So,(Ravi + Rishi + Ronit)s 1 days work = 1/20
So, they all together can finish the work in 20 days.
Ravis 1 days work = [(Ravi + Rishi + Ronit)s 1 days work] [(Rishi + Ronit)s 1 days work]
= (1/20 1/30) = 1/60
So, Ravi alone can finish the work in 60 days.
Similarly, Rishis 1 day work = (1/20 1/40) = 1/40
So, Rishi alone can finish the work in 40 days
And Ronits 1 day work = (1/20 1/24)
= 1/120
Numerical Aptitude Number System
For Any Guidance Call Our Expert at +91 8800734161

Click Here For Hard Copy of this Study Materials:
http://bankpoclerk.com/community/study-kit/ibps-po-capsule
So, Rishi alone can finish the work in 120 days.
Numerical Aptitude Time and Work
2013 bankpoclerk.com
1. A and B together can do a piece of work in 12
days, which B and C together can do in 16
days. After A has been working at it for 5 days
and B for 7 days, C finishes it in 13 days. In
how many days C alone will do the work?
(1) 16 (2) 24
(3) 37 (4) 48
(5) None of these
2. A and B can do a piece of work in 45 days and
40 days respectively. They began to do the
work together but A leaves after some days
and then B completed the remaining work in
23 days. The number of days after which A
left the work was:
(1) 11 (2) 7
(3) 9 (4) 12
(5) None of these
3. A can do a piece of work in 14 days which B
can do in 21 days. They begin together but 3
days before the completion of the work, A
leaves off. The total number of days to
complete the work is:
(1)
3
6
5
(2)
1
8
2
(3)
1
10
5
(4)
1
13
2
(5) None of these
4. A, B and C can complete a work separately in
24, 36 and 48 days respectively. They started
together but C left after 4 days of start and A
left 3 days before the completion of the work.
In how many days will the work be completed?
(1) 15 days (2) 24 days
(3) 25 days (4) 38 days
(5) None of these

Numerical Aptitude Number System


For Any Guidance Call Our Expert at +91 8800734161

Click Here For Hard Copy of this Study Materials:
http://bankpoclerk.com/community/study-kit/ibps-po-capsule
ANSWERS
1. (2) 2. (3) 3. (3) 4. (1)
EXPLANATIONS
1. As 5 days work + Bs 7 days work + Cs 13
days work = 1
(A + B)s 5 days work + (B + C)s 2 days
work + Cs 11 days work = 1
+ Cs 11 days work = 1

5 2
12 16
+
+ Cs 11 days work = 1.
Cs 11 days work = 1
5 2
12 16
_
+

,
=
11
24
.
Cs 1 days work =
11 1 1
24 11 24
_


,
.
C alone can finish the work in 24 days.
2. (A + B)s 1 days work =
1 1 17
45 40 360
_
+

,
.
Work done by B in 23 days =
1 23
23
40 40
_


,
.
Remaining work =
23
1
40
_


,
=
17
40
.
Now,
17
360
work was done by (A + B) in 1 day.
17
40
work was done by (A + B) in
360 17
1
17 40
_


,
=
9 days.
A left after 9 days.
3. Bs 3 days work =
1 1
3
21 7
_


,
.
Remaining work =
1 6
1
7 7
_


,
(A + B)s 1 days work =
1 1 5
14 21 42
_
+

,
.
Now,
5
42
work is done by A and B in 1 day.

6
7
work is done by A and B in =
42 7
5 7
_


,
=
36
5
= days.
Hence, total time taken =
36
3
5
_
+

,
days =
1
10
5
days.
Numerical Aptitude Pipes and Cisterns
2013 bankpoclerk.com
1. A pump can fill a tank with water in 2 hours. Because
of a leak, it took
1
2
3
hours to fill the tank. The leak can
drain all the water of the tank in:
(1) 43 hrs (2) 9 hrs
(3) 10 hrs (4) 14 hrs
(5) None of these
2. Two taps A and B can fill a tank in 5 hours and 20 hours
respectively. If both the taps are opened then due to a
leakage, it took 30 minutes more to fill the tank. If the
tank is full, how long will it take for the leakage alone to
empty the tank?
(1) 44 hrs (2) 12 hrs
(3) 18 hrs (4) 36 hrs
(5) None of these
3. Two pipes A and B together can fill a cistern in 4 hours.
Had they been opened separately, then B would have
taken 6 hours more than A to fill the cistern. How much
time will be taken by A to fill the cistern separately?
(1) 10 hr (2) 4 hrs
(3) 6 hrs (4) 8 hrs.
(5) None of these
4. One pipe can fill a tank three times as fast as another
pipe. If together the two pipes can fill the tank in 36
minutes, then the slower pipe alone will be able to fill
the tank in:
(1) 92 min (2) 112 min
(3) 144 min (4) 192 min
(5) None of these
17
Pipes and Cisterns
ANSWERS
1. (4) 2. (4) 3. (3) 4. (3)
EXPLANATIONS
1. Work done by the leak in 1 hour =
1 3
2 7
_


,
=
1
14
Leak will empty the tank in 14 hrs.
2. Part filled by (A + B) in 1 hour =
1 1
5 20
_
+

,
=
1
4
So, A and B together can fill the tank in 4 hours.
Work done by the leak in 1 hour =
1 2
4 9
_


,
=
1
36
Leak will empty the tank in 36 hrs.
3. Let the cistern be filled by pipe A alone in x hours.
Then, pipe B will fill it in (x + 6) hours.
x
2
2x 24 = 0
(x 6) (x + 4) = 0
x = 6.
[neglecting the ve value of x]
4. Let the slower pipe alone fill the tank in x minutes.
Then, faster pipe will fill it in
3
x
minutes.
1 3
x x
+
=
1
36

4
x
=
1
36
x = 144 min.
Numerical Aptitude Time and Distance
2013 bankpoclerk.com
-
Distance covered, time and speed are related by
Time =
Distance
Speed
...(i)
Speed =
Distance
Time
...(ii)
Distance = Speed Time ...(iii)
Distance is measured in metres, kilometres and miles.
Time in hours, minutes and seconds.
Speed in km/h, miles/h and m/s.
1. To convert speed of an object from km/h to m/s multiply the speed by
5
18
.
2. To convert speed of an object from m/s to km/h, multiply the speed by
18
5
.

It is the ratio of total distance covered to total time of journey.
Average speed =
Total distance covered
Total time of journey
- - -
Rul e 1
If a certain distance is covered with a speed of x km/h and another equal distance with a speed of y
km/h, then the average speed for the whole journey is the harmonic mean of the two speeds.
Average speed =
2
km/h
1 1
x y
_



+

,
=
2
km/h
xy
x y
_

+
,
18
Time and Distance
Numerical Aptitude Time and Distance
2013 bankpoclerk.com
Rul e 2
If three equal distances are covered by three different speeds x, y and z km/h, then average speed for
the whole journey is given by
Average speed =
3
km/h
1 1 1
x y z
_



+ +

,
=
3
km/h
xy yz zx
_

+ +
,
Rul e 3
If a certain distance is covered with a speed of x km/h and another distance with a speed of y km/
h but time interval for both journeys being same, then average speed for the whole journey is given by
Average Speed =
km/h
2
x y + _

,
Rul e 4
If a certain distance is covered with a speed of x, y and z km/h, but time inverval for the three journey
being equal, then average speed is given by
Average speed =
km/h
3
x y z + + _

,
Rul e 5
If the ratio of speeds A and B is x : y, then the ratio of times taken by them to cover the same distance
is
1 1
: .
x y

(i) If two bodies are moving in the same direction at x km/h and y km/h, where (x > y), then their
relative speed is given by (x y) km/h.
(ii) If two bodies are moving in opposite direction at x km/h and y km/h, then their relative speed is
given by (x + y) km/h.
- -
Rul e 1 Tr ai n Vs Stati onar y Obj ects of no Length
Time taken by a train of length l metre to pass a stationary object such as a pole, standing man or a
building is equal to the time taken by the train to cover l metre.
Speed of the train =
Length of the train
Time taken to cross the stationary object
Rul e 2 Tr ai n Vs Stati onar y Obj ects of Cer tai n Length
Time taken by a train of length l metre to pass a stationary object of length a metre such as another
standing train, bridge or railway platform is equal to the time taken by the train to cover (l + a) metre.
Speed of the train =
Length of the train+Lengthof the stationary object
Time taken to cross the stationary object
Numerical Aptitude Number System
For Any Guidance Call Our Expert at +91 8800734161

Click Here For Hard Copy of this Study Materials:
http://bankpoclerk.com/community/study-kit/ibps-po-capsule
Numerical Aptitude Time and Distance
2013 bankpoclerk.com
Rul e 3 Tr ai n Vs Movi ng Obj ects of no Length
Time taken by the train of length l metre to pass a man moving is equal to the time taken by the
train to cover l metre
(i) When the train and man move in the same direction with speeds of x m/s and y m/s. Then,
(x y) =
Length of the train
Time taken to cross each other
(ii) When the train and man move in opposite directions with speeds of x m/s and y m/s. Then,
(x + y) =
Length of the train
Time taken to cross each other
Rul e 4 Tr ai n Vs Movi ng Obj ects of Cer tai n Length
Time taken by the train of length l metre to pass a moving object of length a metre such as another
moving train is equal to the time taken by the train to cover (l + a) metre.
(i) When the two trains move in the same direction with speeds of x m/s and y m/s, (x > y), then
(x y) =
Length of the train + Length of train two
Time taken to cross each other
(ii) When the two tains move in opposite directions with speeds of x m/s and y m/s. Then,
(x + y) =
Length of the train one + Length of train two
Time taken to cross each other
Rul e 5 Two Movi ng Tr ai ns
If two trains start at the same time from points A and B towards each other and after crossing they
take a and b second in reaching B and A respectively. Then, (As speed) : (Bs speed) =
: b a
.
- - -
-
When an object is moving in their direction in which the water in the stream is flowing, then the
object is said to be downstream.
-
When an object is moving against (opposite) direction in which the water in the stream is flowing,
then the object is said to be moving upstream.

When an object is moving in water where there is no motion in water, the object can move in any
direction with a uniform speed, then the object is said to be moving in still water.
Rul e 1 Downstr eam and Upstr eam Speed
Let the speed of the boat in still water = x km/h and speed of the stream be y km/h, then
Speed of the boat with stream downstream speed = (x + y) km/h
Speed of the boat against stream = upstream speed = (x y) km/h
As, when the boat is moving downstream, the speed of the water aids the speed of the boat and when
the boat is moving upstream, the speed of the water reduces the speed of the boat.
Numerical Aptitude Number System
For Any Guidance Call Our Expert at +91 8800734161

Click Here For Hard Copy of this Study Materials:
http://bankpoclerk.com/community/study-kit/ibps-po-capsule
Numerical Aptitude Time and Distance
2013 bankpoclerk.com
Rul e 2 Speed of Boat i n Sti l l Water & Speed of Str eam
If the downstream speed of boat is a km/h and the upstream speed of boat is b km/h, then
Speed of boat in still water =
1
( )km/h
2
a b +
Speed of stream =
1
( )km/h
2
a b
- -
Rul e 1
When two people are running around a Circular Track starting at the same point and at the same
time, then whenever the two people meet the person moving with a greater speed covers one round
more than the person moving with lesser speed.
Rul e 2
When two people with speeds of x km/h and y km/h start at the same time and from the same point
in the same direction around a circular track of circumference c km, then
The time taken to meet for the first time anywhere on the track =
h
c
x y
The time taken to meet for the first time at the starting point = LCM of
,
c c
h
x y
_

,
Rul e 3
When two people with speeds of x km/h and y km/h respectively start at the same time and from the
same point but in opposite direction around a circular track of circumference c km, then
The time taken to meet for the first time anywhere on the track =
h
c
x y +
The time taken to meet for the first time at the starting point = LCM of
, h
c c
x y
_

,
Example 1: Convert 90 km/h into m/s.
Sol uti on. 90 km/h =
_


,
5
90 m/s
18
= 25 m/s
Example 2: Convert 10 m/s into km/h.
Sol uti on. 10 m/s =
18
10 m/s
5
_


,
= 36 km
Example 3: A man can cover a certain distance in 1 h 30 min by covering one-third of the distance at
6 km/h and the rest at 15 km/h. Find the total distance.
Sol uti on. Let the total distance be x km. Then,
2
3 3
6 15
x x
+
=
3
2

2
18 45
x x
+
=
3
2

9
90
x
=
3
2

10
x
=
3
2
Numerical Aptitude Number System
For Any Guidance Call Our Expert at +91 8800734161

Click Here For Hard Copy of this Study Materials:
http://bankpoclerk.com/community/study-kit/ibps-po-capsule
Numerical Aptitude Time and Distance
2013 bankpoclerk.com
x =
(3 10)
2

= 15
Total distance = 15 km
Example 4: An aeroplane started one hour later than the scheduled departure from a place 1200 km
away from its destination. To reach the distination on time, the pilot had to increase its speed by 200 km/
h. What was the normal speed of the aeroplane?
Solution. Let the time taken by the aeroplane in second case be x hour. Then,
1200
x
=
1200
200
1 x
+
+

6
x
=
6
1
1 x
+
+
6x + 6 = 6x + x
2
+ x x
2
+ x 6 = 0
(x + 3) (x 2) = x x = 2h (Q = 3 is not possible)
Time taken in second case = 2 h
So, Speed =
1200
2
= 600 km/h
Hence, normal speed = 600 200 = 400 km/h
Numerical Aptitude Number System
For Any Guidance Call Our Expert at +91 8800734161

Click Here For Hard Copy of this Study Materials:
http://bankpoclerk.com/community/study-kit/ibps-po-capsule

1. Mac travels from A to B a distance of 250 miles


in 5 hours. He returns to A in 4 hours 30
minutes. His average speed is:
(1) 42 mph (2) 49 mph
(3) 48 mph (4) 50 mph
(5) None of these
2. A boy goes to his school from his house at a
speed of 3 km/hr and returns at a speed of 2
km/hr. If he takes 5 hours in going and coming,
the distance between his house and school is:
(1) 8.5 km (2) 5.5 km
(3) 6 km (4) 9 km
(5) None of these
EXPLANATIONS
1. Speed from A to B =
2
250
11
_


,
mph
=
500
11
_

,
mph.
Speed from B to A =
2
250
9
_


,
mph
=

500
9
_

,
mph.
Average speed =
500 500
2
11 9
500 500
11 9
_



+
,
mph
=
500000
4500 5500
_

, +
mph = 50 mph.
2. Average speed =
2 3 2
3 2
_

, +
km/hr =
12
5
km/hr.
Distance travelled =
12
5
5
_


,
km = 12 km.
Distance between house and school
=
12
2
_

,
= 6 km.
1. A train moves past a phone post and a bridge 264 m
long in 8 seconds and 20 seconds respectively. What is
the speed of the train?
(1) 75 km/hr (2) 82 km/hr
(3) 79 km/hr (4) 79.2 km/hr
(5) None of these
2. A train takes 18 seconds to pass completely through a
station 162 m long and 15 seconds through another station
120 m long. The length of the train is:
(1) 73 m (2) 92 m
(3) 90 m (4) 100 m
(5) None of these
3. How many seconds will a 500 metre long train take to
cross a man walking with a speed of 3 km/hr in the
direction of the moving train if the speed of the train is
63 km/hr?
(1) 32 (2) 30
(3) 40 (4) 48
(5) None of these
4. A jogger running at 9 kmph alongside a railway track is
240 metres ahead of the engine of a 120 metre long
train running at 45 kmph in the same direction. In how
much time will the train pass the jogger?
(1) 38 sec (2) 20 sec
(3) 36 sec (4) 72 sec
(5) None of these
19
Problems on Trains
ANSWERS
1. (4) 2. (3) 3. (2) 4. (3)
1. Let the length of the train be x metres and its speed by
y m/sec.
They,
x
y
= 8 x = 8y
Now,
264
20
x +
= y 8y + 264 = 20y
y = 22.
Speed = 22 m/sec
=
18
22
5
_


,
km/hr = 79.2 km/hr..
2. Let the length of the train be x metres.

162
18
x +
=
120
15
x +
15 (x + 162) =
18 (x + 120) x = 90 m.
3. Speed of train relative to man = (63 3) km/hr = 60 km/
hr
=
5
60
18
_


,
m/sec =
50
3
m/sec.
Time taken to pass the man
=
3
500
50
_


,
sec = 30 sec.
4. Speed of train relative to jogger
= (45 9) km/hr = 36 km/hr
=
5
36
18
_


,
m/sec = 10 m/sec.
Distance to be covered
= (240 + 120) m = 360 m.
Time taken=
360
10
_

,
sec = 36 sec.
2013 bankpoclerk.com
Numerical Aptitude Boats and Streams
1. Speed of a boat in standing water is 9 kmph and the
speed of the stream is 1.5 kmph. A man rows to a place
at a distance of 105 km and comes back to the starting
point. The total time taken by him is:
(1) 22 hours (2) 27 hours
(3) 20 hours (4) 24 hours
(5) None of these
2. The speed of a boat in still water is 15 km/hr and the
rate of current is 3 km/hr. The distance travelled
downsteam in 12 minutes is:
(1) 3.3 km (2) 2.9 km
(3) 2.4 km (4) 3.6 km
(5) None of these
3. A man can row at 5 kmph in still water. If the velocity
of current is 1 kmph and it takes him 1 hour to row to a
place and come back, how far is the place?
(1) 2.4 km (2) 2.5 km
(3) 3 km (4) 3.1 km
(5) None of these
4. A boat takes 19 hours for travelling downstream from
point A to point B and coming back to a point C midway
between A and B. If the velocity of the stream is 4 kmph
and the speed of the boat in still water is 14 kmph, what
is the distance between A and B?
(1) 162 km (2) 180 km
(3) 223 km (4) 220 km
(5) None of these
20
Problems on Trains
ANSWERS
1. (4) 2. (4) 3. (1) 4. (2)
EXPLANATIONS
1. Speed upstream= 7.5 kmph;
Speed downstream = 10.5 kmph.
Total time taken =
105 105
7.5 10.5
_
+

,
hour
= 24 hours.
2. Speed downstream = (15 + 3) kmph
= 18 kmph.
Distance traveled =
12
18
60
_


,
km
= 3.6 km.
3. Speed downstream = (5 + 1) kmph
= 6 km h;
4. Speed downstream = (14 + 4) km/hr
= 18 km/hr;
Speed upstream (14 4) km/hr
= 10 km/hr.
Let the distance between A and B be x km. Then
( / 2)
18 10
x x
+
= 19

18 20
x x
+
= 19,
19
180
x
= 19
x = 180 km.
Numerical Aptitude Simple I nterest
2013 bankpoclerk.com
-
It is the sum which is paid by the borrower to the lender for using the money for a specific time
period. The money borrowed is called the Principal. The rate at which the interest is calculated on the
principal is called Rate of Interest. The time for which the money is borrowed is Time and the total
sum of principal and interest is called the Amount.
-
If P = Principal, R = Rate per cent per annum T = Number of years, SI = Simple Interest and A =
Amount.
Then,
(i) SI =
100
P T R
(ii) P =
100 SI
R T

(iii) R =
100 SI
P T

(iv) T =
100 SI
P R

(v) A = P + SI = P +
100
P T R
=
1
100
RT
P
_
+

,
Here, the interest is calculated on the original principal ie, the principal to calculate the interest
remains constant throughout the time period. The interest earned on the principal is not taken into
account for the purpose of calculating interest for later years.
Example 1: Find the SI on 7200 at 8% per annum for 10 months.
Sol uti on. Here, P = 7200, R = 8% per annum and T =
10
yr
12
=
5
yr
6
.
SI =
100
P T R _

,
=
5 1
7200 8
6 100
_


,
= 480
Example 2: A sum is lent at 10% per annum. Simple interest will get doubled in how many years?
Sol uti on. Sum will be doubled when SI = P
Therefore, SI = P =
100
PRT
21
Simple Interest
RT = 100 T =
100
R
=
100
10
= 10 years
Numerical Aptitude Simple I nterest
2013 bankpoclerk.com

1. If 64 accounts to 83.20 in 2 years, what


will 86 amount to in 4 years at the same rate
per cent per annum?
(1) 115.80 (2) 127.70
(3) 127.40 (4) 51.60
(5) None of these
2. The simple interest on a certain sum of money
at the rate of 5% p.a. for 8 years is 840. At
what rate of interest the same amount of
interest can be received on the same sum after
5 years?
(1) 10% (2) 8%
(3) 9% (4) 12%
(5) None of these
3. The interest on a certain deposit at 4.5% p.a.
is 202.50 in one year. How much will the
additional interest in one year be on the same
deposit at 5% p.a.?
(1) 30.25 (2) 22.50
(3) 25 (4) 52.75
(5) None of these
4. A sum invested at 5% simple interest per
annum grows to 504 in 4 years. The same
amount at 10% simple interest per annum in
2 years will grow to:
(1) 530 (2) 555
(3) 525 (4) 650
(5) None of these
Numerical Aptitude Number System
For Any Guidance Call Our Expert at +91 8800734161

Click Here For Hard Copy of this Study Materials:
http://bankpoclerk.com/community/study-kit/ibps-po-capsule

ANSWERS
1. (4) 2. (2) 3. (2) 4. (3) 5. (3)
EXPLANATIONS
1. P = 64, S.I. = (83.20 64) = 19.20,
T = 2 years.
So, rate R =
100 19.20
64 2
_

,
% = 15%
Now, P = 86, R = 15%, T = 4 years.
S.I. =
86 15 4
100
_

,
= 51.60.
2. S.I. = 840, R = 5%, T = 8 years.
Principal (P) =
100 840
5 8
_

,
.
= 2100.
Now, P = 2100, S.I. = 840, T = 5 years.
Rate =
100 840
2100 5
_

,
% = 8%
3. S.I. = 202.50, R = 4.5%, T = 1 year.
Principal =
100 202.50
4.5 1
_

,

= 4500.
Now, P = 4500, R = 5%, T = 1 year.
S.I. =
4500 5 1
100
_

,
= 225.
Difference in interest
= (225 202.50) = 22.50.
4. Let the sum be x. Then, S.I. = (504 x).

5 4
100
x _

,
= 504 x
20x = 50400 100x
120x = 50400
x = 420.
Now, P = 420, R = 10%, T =
5
2
years.
S.I. =
420 10 5
100 2
_


,
= 105.
Amount = (420 + 105)
= 525.
2013 bankpoclerk.com
Numerical Aptitude Compound I nterest
In compound interest, the interest is added to the principal at the end of each period and the amount
thus obtained becomes the principal for the next period. The process is repeated till the end of the
specified time.
If P = Principal
R = Rate per cent per unit time
Time = n years
A = Amount; CI = Compound Interest
When the interest is compounded annually
Amount after n years = (1) =
1
100
n
R
P
_
+

,
Compound Interest =
1
100
n
R
P P
_
+

,
=
1 1
100
n
R
P
1
_
1 +

1 ,
]

1. If the rate of interest differs from year to year ie, R
1
in the first year, R
2
in the second year, R
3
in
the third year. Then, A =
3 1 2
1 1 1
100 100 100
R R R
P
_ _ _
+ + +

, , ,
2. When the principal changes every year, we say that the interest is compounded annually. Then,
A =
1
100
n
R
P
_
+

,
3. When the principal changes as per every six months, we say that the interest is compounded half
yearly or semi-annually. Then,
A =
2
2
1
100
n
R
P
_

+



,
22
Compound Interest
Numerical Aptitude Compound I nterest
2013 bankpoclerk.com
4. When the principal changes every three months, we say that the interest is compounded quarterly.
Then,
A =
4
4
1
100
n
R
P
_

+



,
5. When the principal changes after every month, we say that the interest is compounded monthly.
Then,
A =
12
12
1
100
n
R
P
_

+



,
6. When the interest is compounded annually but time is in fraction say
3
2
4
year.
Then, A =
2
3
4
1 1
100 100
R
R
P
_

_
+ +


,


,
7. The difference between the simple interest and compound interest for 2 year (or terms) is given by
the formula
D =
2
100
R
P
_

,
Where D is the difference, P is the principal and R is the rate of interest.
8. Present worth of x due n years, hence is given by
Present worth =
1
100
n
x
R _
+

,
Example 1: Find the compound interest on 5500 at 9% per annum for 2 year, if the interest is
compounded annually?
Sol uti on. P = 5500, R = 10% per annum and n = 2 year
AI 5500
2
8
1
100
_
+

,
= 5500
109 109
100 100

= Rs. 6534.56
CI = A P = 6534.55
= Rs. 1034.55
Numerical Aptitude Number System
For Any Guidance Call Our Expert at +91 8800734161

Click Here For Hard Copy of this Study Materials:
http://bankpoclerk.com/community/study-kit/ibps-po-capsule
Numerical Aptitude Compound I nterest
2013 bankpoclerk.com
EXERCISE
1. The difference between compound interest
and simple interest on an amount of 15,000
for 2 years is 96. What is the rate of interest
per annum?
(1) 8 (2) 11
(3) 12 (4) 13
(5) None of these
2. The difference between simple and compound
interests and compounded annually on a
certain sum of money for 2 years at 4% per
annum is Re. 1. The sum (in ) is:
(1) 625 (2) 620
(3) 640 (4) 660
(5) None of these
3. The compound interest on a sum of money for
2 years is 832 and the simple interest on the
same sum for the same period is 800. The
difference between the compound interest
and the simple interest for 3 years will be:
Numerical Aptitude Number System
For Any Guidance Call Our Expert at +91 8800734161

Click Here For Hard Copy of this Study Materials:
http://bankpoclerk.com/community/study-kit/ibps-po-capsule
EXPLANATIONS
1.
2
15000 R 2
15000 1 15000
100 100
R
1
_ _
1 +

, 1 ,
]
= 96
15000
2
2
1 1
100 100
R R
1
_
+
1

,
1
]
= 96

2
(100 ) 10000 200
15000
10000
R R 1 +
1
]
= 96
R
2
=
96 2
3

= 64
R = 8.

Rate = 8%
2. Let the sum be x. Then,
C.I. =
2
4
1
100
x x
1
_
+
1

,
1
]
=
676
'
625
x x
_


,
=
51
625
x.
S.I. =
4 2
100
x _

,
=
2
25
x
.

51 2
625 25
x x

= 1 or x = 625.
3. Difference in C.I. and S.I. for 2 years
= 32.
S.I. on 400 for one year = 32.
So, Rate =
100 32
400 1
_

,

% = 8%.
Hence, difference in C.I. and S.I. for 3
rd
year
= S.I. on 832 =
832 8 1
100
_

,
= 66.56.
Total difference = (32 + 66.56)
= 98.56.
Numerical Aptitude Compound I nterest
2013 bankpoclerk.com
Area
The area of a plane figure is the measure of the surface enclosed by its boundary. The area of a
triangle or a polygon is the measure of the surface enclosed by its sides.
1. Triangle
Area of the triangle =
1
Base Height
2
_


,
=
1
2
bh
Area of the triangle, = ( )( )( ) s s a s b s c
where s =
1
( )
2
a b c + +
Perimeter of the triangle = a + b + c = 2s
Radius of incircle of a triangle = /s
2. Right-Angles Triangle
Area =
1
2
bh
Hypotenuse, d =
2 2
b h +
(Pythagoras theorem)
Perimeter = b + d + h
3. Isosceles Right-Angled Triangle
Area =
1
( )
2
a a
=
2
2
a
Hypotenuse, d =
2 2
a a +
=
2a
Perimeter = 2a +
2a
A
B C
a
b
c
h
A
B C
h d
b
A
B C
a d
a
23
Area of Plane Figures
Numerical Aptitude Area of Plane Figures
2013 bankpoclerk.com
4. Equilateral Triangle
AD = h =
2
2
2
a
a
_


,
=
2
2
4
a
a
h =
2
3
4
a
=
3
2
a
Area =
1
2
bh
=
1 3
2 2
a
a =
2
3
4
a
or
2
3
h
Perimeter = 3a
Radius of incircle of an equilateral triangle of side a =
2 3
a
Radius of circumcircle of an equilateral triangle of side a =
3
a
5. Square
Area = (Side)
2
= a
2
=
2
1
(diagonal)
2

Diagonal =
2(side)
=
2a
Perimeter = 4a
6. Rectangle
Area = Length Breadth = l b
Diagonal =
2 2
l b +
Perimeter = 2 (l + b)
Area of 4 walls = 2 (l +b) h
7. Parallelogram
Opposite sides are parallel and equal and opposite angles are equal.
Area = Base Height = b h
Perimeter = 2 (a + b)
a
D
A
B C
h a
2
a
2
a
D
A B
C
b
I
D
A
C
B
a
h
b
A B
C D
a
a
Numerical Aptitude Number System
For Any Guidance Call Our Expert at +91 8800734161

Click Here For Hard Copy of this Study Materials:
http://bankpoclerk.com/community/study-kit/ibps-po-capsule
2013 bankpoclerk.com
Numerical Aptitude Area of Plane Figures
8. Rhombus
All sides are equal and opposite angles are equal. Diagonals bisect
each other at right angles.
Area =
1 2
1
2
d d
Side =
2 2
1 2
4 4
d d
+
=
2 2
1 2
1
2
d d +
Perimeter = 4a
9. Trapezium
Area =
1
2
(Sum of parallel sides) (Distance between them)
=
1
( )
2
a b h +
10. Quadrilateral
Area =
1 2
1 1
2 2
h d h d
+

=
1 2
1
( )
2
d h h +
11. Circle
r = Radius
d = Diameter = 2r
Circumference =
22
2
7
r
_


,
12. Semi-Circle
Area =
2
2
r
Circumference =
22
7
r
_


,
13. Sector
r = Radius, l = Length of the arc
= Central angle
A
a
a
B D
C
a a
d1
d2
D a C
B
b
A
h
D
h1
A
E
d
F
C
B
h2
r
O
r
O
I
r r

Numerical Aptitude Number System


For Any Guidance Call Our Expert at +91 8800734161

Click Here For Hard Copy of this Study Materials:
http://bankpoclerk.com/community/study-kit/ibps-po-capsule
Numerical Aptitude Area of Plane Figures
2013 bankpoclerk.com
Area of the sector =
1
(Arc length )
2
r
=
2
360
r
where arc length =
2
360
r
Example 1: One side of a rectangular field is 16 m and one of its diagonal is 20 m. Find the area of
the field.
Sol uti on. BC
2
= AC
2
AB
2
= (20)
2
(16)
2
= 400 256 = 144
A B
C D
16 m
BC = 12m
Area = (l b) = (16 12)m
2
= 192 m
2
Example 2: The length and breadth of a room are in the ratio of 3 : 2. Its area is 864 m
2
. Find its
perimeter.
Sol uti on. Let the length be 3x m and breadth be 2x m.
Area = (3x 2x) = 6x
2
= 864 x
2
= 144 x = 12
Length = 3x = (3 12) m = 36 m
Breadth = 2x = (2 12) m = 24 m
Perimeter = 2 (l + b) = 2 (36 + 24) = 120 m
Example 3: Find the circumference of a circle whose diameter is 14 m.
Sol uti on. r =
diameter
2
=
14
2
= 7m
Circumference = 2r =
22
2 7
7

= 44m
Example 4: The areas of two concentric circles are 154m
2
and 308m
2
respecively. Find the width of
the ring made by them.
Sol uti on. The radius of inner circle r =
154

= 7m
Another circle is outside this circle.
If the width of the ring be then area of the ring
(2r + ) = 308 154

22
(14 )
7
+
= 154
= 289 m
Numerical Aptitude Number System
For Any Guidance Call Our Expert at +91 8800734161

Click Here For Hard Copy of this Study Materials:
http://bankpoclerk.com/community/study-kit/ibps-po-capsule
2013 bankpoclerk.com
Numerical Aptitude Area of Plane Figures
1. The diagonal of a rectangle is thrice its
smaller side. The ratio of the length to the
breadth of the rectangle is:
(1) 4 : 1 (2) 3 :1
(3) 2 :1 (4) 2 2 :1
(5) None of these
2. A rectangular carpet has an area of 120 sq.
metres and a perimeter of 46 metres. The
length of its diagonal is:
(1) 15 m (2) 19 m
(3) 17 m (4) 22 m
(5) None of these
3. The diagonal of a rectangle is 41 cm and its
area is 20 sq. cm. The perimeter of the
rectangle must be:
(1) 10 cm (2) 18 cm
(3) 22 cm (4) 42 cm
(5) None of these
4. A took 15 seconds to cross a rectangular field
diagonally walking at the rate of 52 m/min and
B took the same time to cross the same field
along its sides walking at the rate of 68 m/
min. The area of the field is:
(1) 52 m
2
(2) 40 m
2
(3) 61 m
2
(4) 60 m
2
(5) None of these
EXERCISE
Numerical Aptitude Number System
For Any Guidance Call Our Expert at +91 8800734161

Click Here For Hard Copy of this Study Materials:
http://bankpoclerk.com/community/study-kit/ibps-po-capsule
ANSWERS
1. (4) 2. (3) 3. (2) 4. (4)
EXPLANATIONS
1.
2 2
l b +
= 3b l
2
+ b
2
= 9b
2
8b
2
l
2

2
2
l
b
= 8
l
b
= 8 = 2 2 .
2. 2(l + b) = 46 or l + b = 23.
Also, lb = 120.
Diagonal =
2 2 t
l b
=
2
( ) 2 l b lb +
=
2
(23) 240 = 17 m.
3.
2 2
l b + = 41 or l
2
+ b
2
= 41.
Also, lb = 20.
(l + b)
2
= (l
2
+ b
2
) + 2lb
= 41 + 40 = 81
(l + b) = 9.
Perimeter = 2 (l + b) = 18 cm.
4. Length of diagonal =
15
52
60
_


,
= 13 m.
Numerical Aptitude Volume and Surface Area of Solid Figures
2013 bankpoclerk.com
Solid
Anything that occupies space is called a Solid. In addition to area, a solid figure has volume also. It
has three dimensions namely, length, breadth and height. For solid two different types of areas namely,
lateral surface area or curved surface area and total surface area are defined.
1. Prism
A solid having two congruent and parallel faces, called bases and whose other faces, the lateral faces
are parallelograms, formed by joining corresponding vertices of the bases is called a Prism.
2. Right Prism
A prism in which bases are perpendicular to the lateral edges is called a Right Prism. The base of
the prism can be a polygon.
In a right prism
(i) Number of lateral surfaces = Number of sides of the base of the prism
(ii) Total number of surfaces of a prism = Number of lateral surfaces + 2
(iii) Lateral surface area = Perimeter of base Height
(iv) Total surface area = Lateral surface area + 2 (Area of base)
(v) Volume = Area of base Height
3. Cuboid
A right prism in which the base is a rectangle is called a Cuboid. If l is the length and b the breadth
of the base and h the height, then
h
I
b
Lateral surface area = 2 (l + b)h sq unit
24
Volume and
Surface Area of Solid Figures
2013 bankpoclerk.com
Numerical Aptitude Volume and Surface Area of Solid Figures
Total surface area = 2(l + b)h + 2lb = 2 (lb + bh + lh) sq unit
Volume = lbh cu unit
The longest diagonal of the cuboid =
2 2 2
l b h + +
unit
4. Cube
A right prism in which the base is a square and height is equal to the side of the base is alled a cube.
If x is the side of the cube, then lateral surface area = 4x



x = 4x
2
sq unit
Total surface area = 4x
2
+ 2(x
2
) = 6x
2
sq unit
Volume = x
2
x = x
3
cu unit
The longest diagonal of the cube =
3x
unit
5. Cylinder
n
r
If the base of a right prism is circular, then it becomes a cylinder. The lateral surface area c the
cylinder is a single curved surface called the curved surface area. If r is the radius of the base and h is the
height of the cylinder, then
Curved surface area = 2rh sq unit
Total surface area = 2rh + 2r
2
= 2r (r + h) sq unit
Volume of cylinder = r
2
h cu unit
6. Cone
h I
O r
If the base of a right pyramid is circular, then it becomes a cone. The lateral surface area of the cone
Numerical Aptitude Number System
For Any Guidance Call Our Expert at +91 8800734161

Click Here For Hard Copy of this Study Materials:
http://bankpoclerk.com/community/study-kit/ibps-po-capsule
Numerical Aptitude Volume and Surface Area of Solid Figures
2013 bankpoclerk.com
is a single curved surface. If r is the radius of the base h is the height of the cone and l the slant height of
the cone, then slant height,
2 2
l h r + unit
Curved surface area = rl sq unit
Total surface area = rl + r
2
= r (l + r) sq unit
Volume of cylinder =
1
3
r
2
h cu unit.
7. Frustrum of a Cone
r
h
R
If a cone is cut parallel to base at a hight h then the remaining part is called the frustrum of the cone.
If radius of base and vertex of the frustrum of a cone be R and r respectively and height and slant
height of it be h and l then,
Area of slant part = (R + r) sq.units
where l
2
= h
2
+ (R r)
2
Total surface area = R
2
+ r
2
+ l(R + r) = [R
2
+ r
2
+ l(R + r)] sq.unit
Volume =
1
3
h [R
2
+ r
2
+ Rr] cu unit
8. Sphere
O
r
A sphere is a solid in which any point on the surface of sphere is equidistant from the centre of the
sphere.
Surface area = 4r
2
sq unit
Volume =
4
3
r
3
cu unit
9. Hemisphere
r
O
When a sphere is cut into two equal halves, each half is an hemisphere in which the base is circular
with radius r and height is equal to the radius.
Curved surface area = 2r
2
sq. unit
Numerical Aptitude Number System
For Any Guidance Call Our Expert at +91 8800734161

Click Here For Hard Copy of this Study Materials:
http://bankpoclerk.com/community/study-kit/ibps-po-capsule
2013 bankpoclerk.com
Numerical Aptitude Volume and Surface Area of Solid Figures
Total surface area = 2r
2
+ r
2
sq. unit = 3r
2
sq. unit
Volume = 2r
3
cu unit
1 L = 1000cm
3
10. Pyramids
O
A
B C
D
E F
If a point O is joined to the end points of each side of a polygon by straight lines, then it is called a
pyramid.
Slant surface area =
1
2
Slant height Base side
Total surface area = Base area + Slant area
Volume =
1
3
Base area Height
Base area depends on the shape of polygon in the base.
Example 1: Find the volume and total surface area of a cube of side 7 m.
Sol uti on. Here, x =7 m
Surface area = (6 7
2
) m
2
= 294 m
2
Volume = x
3
= (7
3
) m
3
= 343 m
3
Example 2: Find the volume and total surface area of a cuboid 10 m long, 6 m broad and 4 m high.
Sol uti on. Here, l =10 m, b = 6 m and h = 4 m
Volume = lbh = (10 6 4) m
3
= 240 m
3
Surface area = 2 (lb + bh + lh) = 2 [(10 6) + (6 4) + (4 10)] m
2
= 2 [60 + 24 + 40] = 248 m
2
Example 3: Find the length of the longest pole that can be put in a room 15 m long, 8 m broad and
7 m high.
Sol uti on. Here, l = 15 m, b = 8 m and h = 7 m
Numerical Aptitude Number System
For Any Guidance Call Our Expert at +91 8800734161

Click Here For Hard Copy of this Study Materials:
http://bankpoclerk.com/community/study-kit/ibps-po-capsule
2013 bankpoclerk.com
Numerical Aptitude Volume and Surface Area of Solid Figures
EXERCISE
1. A rectangular water tank is 80 m 40 m.
Water flows into it through a pipe 40 sq. cm
at the opening at a speed of 10 km/hr. By how
much, the water level will rise in the tank in
half an hour?
(1)
4
11
cm (2)
5
9
cm
(3)
5
8
cm (4)
4
5
cm
(5) None of these
2. A hall is 15 m long and 12 m broad. If the sum
of the areas of the floor and the ceiling is equal
to the sum of areas of the four walls, the
volume of the hall is:
(1) 720 (2) 800
(3) 1200 (4) 2000
(5) None of these
3. The sum of the length, breadth and depth of a
cuboid is 19 cm and its diagonal is
5 5
cm. Its
surface area is:
(1) 127 cm
2
(2) 236 cm
2
(3) 361 cm
2
(4) 480 cm
2
(5) None of these
4. A swimming pool 9 m wide and 12 m long is 1
m deep on the shallow side and 4 m deep on
the deeper side. Its volume is:
(1) 309 m
3
(2) 270 m
3
(3) 360 m
3
(4) 607 m
3
(5) None of these
Numerical Aptitude Number System
For Any Guidance Call Our Expert at +91 8800734161

Click Here For Hard Copy of this Study Materials:
http://bankpoclerk.com/community/study-kit/ibps-po-capsule
ANSWERS
1. (3) 2. (3) 3. (2) 4. (2) 5. (2)
EXPLANATIONS
1. Length of water column flown in 1 min.
=
10 1000
m
60
_

,
=
500
m
3
.
Volume flown per minute
=
3
500 40
m
3 100 100
_

,
=
3
2
m
3
.
Volume flown in half an hour =
3
2
30 m
3
_


,
= 20m
3
.
Rise in water level =
20
m
40 80
_

,
=
1
100 cm
160
_


,
=
5
cm
8
.
2. 2(15 + 12) h = 2(15 12)
or h =
180
m
27
=
20
m
3
.
Volume =
3
20
1512 m
3
_

,
= 1200 m
3
.
3. (l + b + h) = 19 and
2 2 2
l b h + + = 5 5 and so
(l
2
+ b
2
+ h
2
) = 125.
Now, (l + b + h)
2
= 19
2
(l
2
+ b
2
+ h
2
) + 2(lb + bh + lh)
= 361 = 2(lb + bh + lh)
= (361 125) = 236.
Surface area = 236 cm
2
.
4. Volume =
3
1 4
12 9 m
2
+ 1 _

1
, ]
= (12 9 2.5)m
2
= 270 m
3
.
Numerical Aptitude Clocks and Calendar
2013 bankpoclerk.com
Clock
A clock has 2 hands, the smaller one is called the hour hand or short hand while the larger one is
called the minute hand or long hand.
The face of a clock is a circle which subtends an angle of 360 at the centre.
Some Important Points
I n every hour
1. (1) Both the hands coincide once. At this point the angle between them is 0..
(2) The hands are straight (point in opposite directions) once. At this point the angles between
them are 180.
(3) The hands are twice perpendicular to each other. At this point the angle between them is 90.
2. (1) In 60 min the minute hand covers 360.
Thus, in 1 min the minute hand covers
360
6
60

_


,
(2) In 12 h the hour hand covers 360.
Thus, in 1 min the hour hand covers
360 1
12 60 2

,
Thus, in 1 minute, the minute hand gains
1 1
6 5 ,
2 2

_


,
than the hour hand.
3. (1) When the two hands are at right angles, they are 15min spaces apart.
(2) When the two hands are in opposite directions, they are 30 min spaces apart.
(3) In 60 min the minute hand gains 55 min on the hour hand.
(4) The minute hand moves 12 times as fast as the hour hand.
4. (1) Too Fast: If a clock indicates 7 : 1.5, when the correct time is 7, it is said to be 15 min too fast.
(2) Too Slow: If a clock indicates 7 : 30, when the correct time is 7 : 45 , it is said to be 15 min too
slow.
25
Clocks and Calendar
2013 bankpoclerk.com
Numerical Aptitude Clocks and Calendar
Calendar
1. Odd Days: The number of days more than the complete weeks for a given period called odd days.
2. Ordinary Year: An ordinary year has 365 days and an ordinary year is not a leap year.
3. Leap Year:
(1) Any year (except a century) which is divisible by 4 is a leap year.
(2) However, every 4th century is a leap year, ie, a century is a leap year when it is divisible by
400.
(3) A leap year has 366 days.
Examples of Leap Year 1924, 1908, 1944, 2008, 1684, etc., are all leap years.
400, 800, 1200, 1600, 2000, etc., are all leap years.
4. Counting of Odd Days
(1) 1 ordinary year = 365 days = (52 weeks + 1 day) 1 ordinary year = 1 odd day
(2) 1 leap year = 366 days = (52 weeks + 2 days) = 1 leap year = 2 odd days
(3) 100 yr = 76 ordinary years + 24 leap years = (76 1 + 24 2) odd days = 124 odd days
= (17 weeks + 5 days) = 5 odd days
(4) 200 yr = (5 2) = 10 odd days = (1 week + 3 days) = 3 odd day
(e) 300 yr = (5 3) = 15 odd days = (2 weeks + 1 day) = 1 odd days
(f) 400 yr = (5 4 + 1) = 21 odd days = (3 weeks + 0 day) = 0 odd day Similarly, 800 yr, 1200 yr, 1600
yr, 2000 yr have 0 odd days.
5. Day of the week with respect to the number of odd days.
Number of odd days Day of the week
0 Sunday
1 Monday
2 Tuesday
3 Wednesday
4 Thursday
5 Friday
6 Saturday
Example 1: Find the angle between the hour hand and the minute hand of a clock when the time is
5 : 35.
Sol uti on. Angle traced by hour hand in 12 h = 360
Angle traced by the hour hand in
35 67 360 67 1
5 h h 167
60 12 12 12 2

_ _


, ,
Angle traced by minute hand in 60 min = 360
Angle traced by minute hand in 35 min =
360
35 210
60

_


,
Required Angle =
1 1
210 167 42
2 2
_


,
Numerical Aptitude Number System
For Any Guidance Call Our Expert at +91 8800734161

Click Here For Hard Copy of this Study Materials:
http://bankpoclerk.com/community/study-kit/ibps-po-capsule
2013 bankpoclerk.com
Numerical Aptitude Clocks and Calendar
EXERCISE
1. A clock is started at 6 Oclock in the morning.
Through how many degrees will the hour hand
rotate when the clock shows 11 Oclock in the
morning?
(1) 140 (2) 180
(3) 150 (4) 160
(5) None of these
2. A clock is started at 5 pm. Through how many
degrees will the hour hand rotate when the
clock shows quarter past 9?
(1) 128.5 (2) 142.5
(3) 127.5 (4) 141.5
(5) None of these
3. At 6 : 15, the hour hand and the minute hand
of a clock form an angle of
(1) 90 (2) 91.5
(3) 97.5 (4) 94.5
(5) None of these
4. The angle between the minute hand and the
hour hand of a clock when the time is 9 : 30, is
(1) 100
o
(2) 180
o
(3) 200
o
(4) 105
o
(5) None of these
Numerical Aptitude Number System
For Any Guidance Call Our Expert at +91 8800734161

Click Here For Hard Copy of this Study Materials:
http://bankpoclerk.com/community/study-kit/ibps-po-capsule
ANSWERS
1. (3) 2. (3) 3. (3) 4. (4)
EXPLANATIONS
1. Angle traced by hour hand in 12 h = 360
Angle traced by hour hand in 5 h
=
360
5 150
12

_


,
2. Angle traced by hour hand in 12 h = 360
Angle traced by hour hand in 4 h 15 min
=
1
4
4
h
=
360 17
127.5
12 4

_


,
3. Angle traced by hour hand in 12 h = 360
Angle traced by hour hand in
25
4
h
=
360 25
187.5
12 4

_


,
Angle traced by minute hand in 60 min = 360
Angle traced by it in 15 min =
360
15
60


,
=90
Required angle = (187.5 90) = 97.5
4. Angle traced by hour hand in 12 h = 360
Angle traced by hour hand in
19
h
2
=
360 19
12 2


,
= 285
Angle traced by minute hand in 60 min = 360
Angle traced by minute hand in 30 min
=
360
30 180
60

_


,
Re uired an le = 285 180 = 105
2013 bankpoclerk.com
Numerical Aptitude Sequences & Series
Arithmetic Progression (AP)
An arithmetic progression is a sequence in which terms increase or decrease by a constant number
called the common difference.
(i) The sequence 2, 6, 10, 14, 18, 22 is an arithmetic progression whose first term is 2 and common
difference 4.
(ii) The sequence
5 7
2, ,3, ,4
2 2
is an arithmetic progression whose first term is 2 and common
difference .
An arithmetic progression is represented by a,(a + d), (a + 2d), (a + 3d) ....... + (n 1)d
Here, a = first term
d = common difference
n = number of terms in the progression
The general term of an arithmetic progression is given by T
n
= a + (n 1) d.
The sum of n terms of an arithmetic progression is given by S, =
2
n
[2a + (n 1) d] or S
n
= 2 [a + l]
where l is the last term of arithmetic progression.
If three numbers are in arithmetic progression, the middle number is called the arithmetic mean
of the other two terms.
If a, b, c are in arithmetic progression, then b =
2
a c +
where b is the arithmetic mean.
Similarly, if n terms a
l
, a
2
, a
3
a
n
are in AP, then the arithmetic mean of these n terms is given
by
AM =
1 2 3
.
n
a a a a
n
+ + ++
If the same quantity is added or multiplied to each term of an AP, then the resulting series is also
an AP.
26
Sequences & Series
Numerical Aptitude Sequences & Series
2013 bankpoclerk.com
If three terms are in AP, then they can be taken as (a d), a, (a + d).
If four terms are in AP, then they can be taken as (a 3d), (a d), (a + d), (a + 3d).
If five terms are in AP, then they can be taken as (a 2d), (a d), a, (a + d), (a + 2d).
Geometric Progression (GP)
A geometric progression is a sequence in which terms increase or decrease by a constant ratio called
the common ratio.
(i) The sequence 1, 3, 9, 27, 81is a geometric progression whose first term is 1 and common ratio 3.
(ii) The sequence
1 1 1 1
1, , , , ,
3 9 27 81

is a geometric progression whose first term is 1 and common ratio
1
.
3
A geometric progression is represented by a, ar, ar
2
ar
n1
.
Here, a = first term
r = common ratio
n = number of terms in the progression.
The general term of a geometric progression is given by T
n
= a
n1
The sum to n terms of a geometric progression is given by
( )
1
,
1
n
n
a r
S
r

when r < 1
( )
1
,
1
n
a r
r

when r > 1
If three numbers are in geometric progression, the middle number is called the geometric mean of
the other two terms.
If a, b, c are in geometric progression, then b ac where b is the geometric mean.
Similarly, if n terms a
1
, a
2
, a
3
, a
4
,a
n
are in geometric progression, then the geometric mean of
these n terms is given by GM = ( )
1
1 2 3
. n
n
a a a a
For a decreasing geometric progression the sum to infinite number of terms is
,
1
a
S
r

where a = first term and r < 1.


If every term of a GP is multiplied by a fixed real number, then the resulting series is also a GP.
If every term of a GP is raised to the same power, then the resulting series is also a GP.
The reciprocals of the terms of a GP is also a GP.
If three numbers are in GP, then they can be taken as
,
a
r
a, ar.
If four numbers are in GP, then they can be taken as
3
3
, , , .
a a
ar ar
r
r
If five numbers are in GP, then they can be taken as
2
2
, , , , .
a a
a ar ar
r
r
Harmonic Progression (HP)
If the reciprocals of the terms of a series form an arithmetic progression, then the series is called a
harmonic progression.
Numerical Aptitude Number System
For Any Guidance Call Our Expert at +91 8800734161

Click Here For Hard Copy of this Study Materials:
http://bankpoclerk.com/community/study-kit/ibps-po-capsule
2013 bankpoclerk.com
Numerical Aptitude Sequences & Series
(i) The sequence
4 3 12
, , ,
3 2 7

is a harmonic progression as
3 2 7
, ,
4 3 12
is in arithmetic progression.
If a, b, c are in harmonic progression, then b =
2ac
a c +
where b is the harmonic mean.
Sum of Natural Series
The sum of the first n natural numbers =
( ) 1
2
n n +
The sum of the square of the first n natural numbers =
( )( ) 1 2 1
6
n n n + +
The sum of the cubes of the first n natural numbers =
( )
2
2
1
4
n n +
The sum of first n even numbers = n(n + 1)
The sum of first n odd numbers = n
2
Example 1: Find the nth term and the fifteenth term of the arithmetic progression 3, 9, 15, 21
Sol uti on. In the given AP we have a = 3, d = (9 3) = 6
T
n
= a + (n 1)d = 3 + (n 1)6 = 6n 3
T
15
= (6 15 3) = 87
Example 2: Find the 10th term of the AP 13, 8, 3, 2,
Sol uti on. In the given AP, we have a = 13, d = (8 13) = 5
T
n
= a + (n 1)d = 13 + (n 1)(5) = 18 5n
T
10
= 18 5 (10) = 32
Example 3: The first term of an AP is -1 and the common difference is -3, the 12th term is
Sol uti on. T
1
= a = 1, d = 3
T
n
= a + (n 1)d = 1 + (n 1)(3) = 2 3n
T
12
= 2 3 12 = 34
Example 4: Which term of the AP 10, 8, 6, 4 is 28?
Sol uti on. We have, a = 10,d = (8 10) = 2, T
n
= 28
T
n
= a + (n 1)d 28 = 10 + (n 1)(2) = n = 20
Example 5: The 8th term of an AP is 17 and the 19th term is 39. Find the 20th term.
Sol uti on. T
8
= a + 7d =17 ...(i)
T
19
= a + 18d = 39 ... (ii)
On subtracting Eq. (i) from Eq. (ii), we get 11d = 22 d = 2
Putting d = 2 in Eq. (i), we get a + 7(2) = 17 = a = (17 14) = 3
First term = 3, Common difference = 2
T
20
= a + 19d = 3 + 19(2) = 41
Numerical Aptitude Number System
For Any Guidance Call Our Expert at +91 8800734161

Click Here For Hard Copy of this Study Materials:
http://bankpoclerk.com/community/study-kit/ibps-po-capsule
2013 bankpoclerk.com
Numerical Aptitude Sequences & Series
EXERCISE
1. If five times the fifth term of an AP is equal to seven times, the seventh term of the AP, then what
is the twelfth term?
(1) 1 (2) 0
(3) 1 (4) 2
(5) None of these
2. Three terms are in AP such that their sum is
18. The sum of the first two terms is six more
than the sum of the last two terms. Find the
last term.
(1) 6 (2) 9
(3) 3 (4) 2
(5) None

of

these
3. Determine k, so that (k + 2), (4k 6) and (3k
2) are three consecutive terms of an AP.
(1) 3 (2) 2
(3) 4 (4) 6
(5) None of these
4. In an AP, the first term is 2 and the sum of
the first five terms is one-fourth the sum of
the next five terms. Find the second term.
(1) 4 (2) 10
(3) 16 (4) 12
(5) None of these
Numerical Aptitude Number System
For Any Guidance Call Our Expert at +91 8800734161

Click Here For Hard Copy of this Study Materials:
http://bankpoclerk.com/community/study-kit/ibps-po-capsule
ANSWERS
1. (2) 2. (3) 3. (1) 4. (1)
EXPLANATIONS
1. T
5
= a + 4d, T
7
= a + 6d
5 (a + 4d) = 7 (a + 6d) 5a + 20d = 7a + 42d
a = 11d
T
12
= a + 11d = 11d + 11d = 0
The twelfth term is 0.
2. Let the three terms be a d, a, a + d.
(a d) + a + (a + d) = 18
3a = 18
a = 6
Also, [(a d) + a] [a + (a + d)] = 6 d = 3
So, the three terms are 9, 6, 3 respectively,
last term = 3.
3. Since, (k + 2), (4k 6) and (3k 2) are in AP.
(4k 6) (k + 2) = (3k 2) (4k 6)
DATA INTERPRETION
Numerical Aptitude (Data I nterpretation) I ntroduction to Data I nterpretation
2013 bankpoclerk.com 2
Contents
1. Introduction to Data Interpretation ............................................................................................. 3
2. Approaches to Data Interpretation ............................................................................................. 6
3. Table Chart ................................................................................................................................... 8
4. Pie Chart ..................................................................................................................................... 53
5. Bar Chart .................................................................................................................................... 83
6. Line Graph................................................................................................................................. 111
7. Mix Diagrams............................................................................................................................. 145
Numerical Aptitude Number System
For Any Guidance Call Our Expert at +91 8800734161

Click Here For Hard Copy of this Study Materials:
http://bankpoclerk.com/community/study-kit/ibps-po-capsule
Numerical Aptitude (Data I nterpretation) I ntroduction to Data I nterpretation
2013 bankpoclerk.com
DATA INTERPRETATION
A Government executives must face a lot of facts and figures in everyday business. Normally these facts are
presented in more compact and precise forms such as
1. Tables (known as data tables)
2. Charts (Pie, Bar, Pert, etc.)
3. Graphs (2D and 3D)
4. Diagrams (Geometric or Venn diagram)
An executives must possess basic skills on deciphering the data from the above mentioned precise forms of
tables, charts etc. which enhances his/ her administrative efficiency. This ability is known as data interpretation.
Example 1
Question: These questions are based on following pie chart, which gives the details of percentage of energy
source of India.
Nuclear, 2%
Hydro, 2%
Gas, 9%
Coal, 51%
Oil, 36%
Q. The second largest energy source of India is
(1) Coal (2) Oil
(3) Gas (4) Hydro and Nuclear
(5) None of these Ans: (2)
1
Introduction to
Data Interpretation
Numerical Aptitude (Data I nterpretation) I ntroduction to Data I nterpretation
2013 bankpoclerk.com
Note: Question may not be always direct. You may need to compute answers before arriving conclusion. Look
the example below.
Example-2:
Question: These questions are based on following table, which gives the details of distribution of paper industry
in India during 1997 especially state wise no of mills, its production capacity in thousand metric ton and percentage
of mills of all India it possess.
State No. of Mills Production Capacity Percentage of
(in 000 MT) all India
Maharashtra 63 624 16.52
Andhra Pradesh 19 427 11.3
Gujarat 55 369 9.77
Uttar Pradesh 68 336 8.89
All India (Total) 379 3778 100
Q. Which of the state has lowest productivity with respect to number of mills it possesses?
(1) Maharashtra (10.19) (2) Andhra Pradesh (22.47)
(3) Gujarat (6.71) (4) Uttar Pradesh (4.94)
(5) None of these Ans: (4)
Note: Also there may be few questions which can not be interpreted using available chart or tables. It needs
careful attention however it is very easy.
Question Patterns of Data Interpretation
There are a multiple ways in which data can be shown to you. They are various forms of tables, charts, bars,
diagrams. For Bank PO exams, you can expect Data Interpretation questions on singular bar, multiple bars, tables,
pie chart, line diagram etc. There you will be provided certain data and instructions. On the basis of the instructions,
you have to answer the questions asked to you, along with the data.
Ways to Prepare for Data Interpretation
For interpreting data, you should read a lot. Read lot of business reports and the analysis given with the
presentation of data. This will give you a fair idea on how data are presented and how to interpret or analyze them.
You must read articles, especially on business which are heavy on data and analyze them. This will keep your DI
cells on toes. In a similar note, refer to the graphs and data provided in various business papers and magazines.
You need to identify techniques to solve questions which require big and time consuming calculations.
At this stage, when there are few months left for Bank PO examyou should solve at least three to four Data
Interpretation sets every day. Also, you need to be good with quick calculation of ratios, percentages, etc. With
consistent practice, you will be able to achieve the same.
While Solving Data Interpretation Questions
While solving the Data Interpretation questions, you must keep some points in mind. Experts suggest the
following points which you should keep in mind while you start attempting the Data Interpretation questions.
Read the title carefully
Take your time and read both the questions and data set carefully. Understand what you are being asked to
do before you begin figuring out the information.
Check the data and types of information required. Be sure that you are looking at the right part of graph or
chart, i.e., on paper columns or rows or proper lines.
Numerical Aptitude Number System
For Any Guidance Call Our Expert at +91 8800734161

Click Here For Hard Copy of this Study Materials:
http://bankpoclerk.com/community/study-kit/ibps-po-capsule
Numerical Aptitude (Data I nterpretation) I ntroduction to Data I nterpretation
2013 bankpoclerk.com
Read the parameters listed along with axes and the scale. This is the most important points as generally one
assumes standard scale as 1 unit =100 or 1 unit = 10. Also the scales of two axes might be different.
The footnotes give the additional information for particular data. This also explains the jargons and lists the
formulae that might be needed to solve the questions asked.
Check the units required. Be sure that your answer is in thousand, millions or whatever it is that the question
specifies.
Numerical Aptitude Number System
For Any Guidance Call Our Expert at +91 8800734161

Click Here For Hard Copy of this Study Materials:
http://bankpoclerk.com/community/study-kit/ibps-po-capsule
Numerical Aptitude (Data I nterpretation) Approaches to Data I nterpretation
2013 bankpoclerk.com
TIPS FOR DATA INTERPRETATION
Here are some hints to remember while solving problems based on data interpretation:
1. Take your time and read carefully. Understand what you are being asked to do before you begin figuring.
2. Check the dates and types of information required. Be sure that you are loking in the proper columns, and on
the proper lines, for the information you need.
3. Check the units required. Be sure that your answer is in thousands, millions, or whatever the question calls
for.
4. Check the units required. Be sure that your answer is in thousands, millions, or whatever the question calls
for.
5. In computing averages, be sure that you add the figures you need and no others, and that you divide by the
correct number of years or other units.
6. Be careful in computing problems asking for percentages.
a. Remember that to convert a decimal into a percent you must multiply it by 100. For example, 0.04 is 4%.
b. Be sure that you can distinguish between such quantities as 1% (1 percent) and 0.01% (one-hundredth of
1 percent), whether in numerals or in words.
c. Remember that if quantity X is greater than quantity Y, and the question asks what percent quantity X is
of quantity Y, the answer must be greater than 100 percent.
7. Dont be afraid of the unfamilliar: Maybe you have never seen such charts before; do not panic and approach
with an open mind.
8. Practice from a lot of sources: Look at different graphs and charts and see how data is represented.
9. Note that there are no purely DI or purely Reasoning Questions. All the questions require some amount of
Reasoning.
10. Look at different newspapers and also the sites on the Internet where you find graphs and charts. An exposure
2
Approaches to
Data Interpretation
Numerical Aptitude (Data I nterpretation) Approaches to Data I nterpretation
2013 bankpoclerk.com
to different types of charts is essential.
11. In Reasoning, do some puzzles from varied sources. Doing these from a variety of sources will help.
12. Do a number of section tests before you start doing full-length tests. Time yourself and stick to the time limits
strictly.
13. Rather than doing conventional sums, rely on logic and develop a reasoning mind. In the exam, there is no
particular type of question that will be repeated, but most questions can be done by using logic.
14. Do thequestions with low weight at the outset. This will give you theconfidence to attempt themore difficult
questions.
15. Develop your strategy by appearing in a number of mock tests. Do not change it at the last moment by listening
to friends or others.
16. Finally, Do Not Panic. Even if you are scoring well in the mock tests, be prepared for something that you may
not have seen before. Learn to keep control even if the paper is formidable; usually it is not.
Numerical Aptitude Number System
For Any Guidance Call Our Expert at +91 8800734161

Click Here For Hard Copy of this Study Materials:
http://bankpoclerk.com/community/study-kit/ibps-po-capsule
Numerical Aptitude (Data I nterpretation) Table Chart
2013 bankpoclerk.com
TABLES
1. Make sure to look at the entiretable or graph.
2. Make sure to express your answer in the correct units (Units of the Tables or Graphs).
3. Look at the possible answers before calculating.
4. Dont confuse decimals and percentages. If the units are percentages, then an entry of .2 means .2% which is
equal to .002.
5. In inference questions, only the information given can be used. See if the answer makes sense.
The table below gives the height and weight of 11 newborn infants.
Height (inches) Weight (pounds)
21 10.2
10 3.3
15 5.6
18 9.0
20 11.0
19 8.2
18 9.5
23 12.1
14 5.0
16 5.9
20 8.9
Q. What is the difference in weight between the tallest and shortest infants?
(1) 13 pounds (1) 12.1 pounds
(3) 8.8 pounds (4) 6.9 pounds
(5) None of these
The correct answer is (3), which is obtained by subtracting 3.3 (the weight of the shortest10 inchesinfant)
from 12.1 (the weight of the tallest23 inchesinfant).
Directions: Study the following table carefully and answer the questions given below.
3
Table Chart
Numerical Aptitude (Data I nterpretation) Table Chart
2013 bankpoclerk.com
1. What is the respective ratio of the number of adult
females to the total number of female children
staying in all the societies together?
(1) 243 : 82 (2) 112 : 71
(3) 82 : 243 (4) 71 : 112
(5) None of these
2. What is the total number of female children staying
in all the societies together?
(1) 314 (2) 433
(3) 410 (4) 343
(5) None of these
3. What is the respective ratio of the total number of
adult males in societies A and B together to the
total number of adult males in societies E and F
together?
(1) 75 : 79 (2) 14 : 17
(3) 79 : 75 (4) 17 : 14
(5) None of these
4. What is the total number of members staying in
all the societies together?
(1) 3520 (2) 3360
(3) 4100 (4) 3000
(5) None of these
5. What is the difference between the number of
male children in society B and the number of male
children in society F?
(1) 84 (2) 14
(3) 96 (4) 26
(5) None of these
1. (1) Number of childrens in
Society A
25
600 150
100
(Total number of Male and Femal es is 600 in
Society A)
Number of female children

60
150 90
100
Number of male children
150 90 60
Society B

40
550 220
100
(Number of children in Society B)
Number of female children

25
220 55
100
Number of male children
220 55 165
Society C

16
575 92
100
(Number of children in Society C)
Number of female children
75
92 69
100
Number of male children
92 69 23
Society D

25
580 145
100
(Number of children in Society D)
Number of female children

20
145 29
100
Number of male children= 145 29 = 116
Society E
40
430 172
100
(Number of children in Society E)
Number of female children=172
50
100
=86
Number of male children= 172 86 =86
Society F
24
1 625 50
100
(Number of children in Society F)
Number of female children=

54
150 81
100
Number of male children= 150 81 =69
Number of adult females=(350 +150 +275 +300 +
250 +300) 410 =1625 410 =1215
Number of female children=90 +55 +69 +29 +86
+ 81 =410
Required ratio =1215 : 410 =243 : 82
2. (3) From question (1)
Number of all female children= 410
3. (4) Number of adult males in Societies A and B =
(250 60) +(400 165) = 190 +235 =425
Number of adult males in Societies E and F =(180
86) +(325 69) = 94 +256 =350
Required ratio = 425 : 350 = 17 : 14
4. (2) Number of all members
=(250 +350) +(400 +150) +(300 +275) +(280 +300)
+(180 +250) +(325 +300) + 150 + 220 + 92 + 145 +
172 + 150 = 4289
5. (3) Required difference = 165 69 = 96
Numerical Aptitude Number System
For Any Guidance Call Our Expert at +91 8800734161

Click Here For Hard Copy of this Study Materials:
http://bankpoclerk.com/community/study-kit/ibps-po-capsule
Numerical Aptitude (Data I nterpretation) Pie Chart
2013 bankpoclerk.com
Circlegraphs are used to show how various sectors share in the whole. Circle graphs are sometimes called pie
charts. Circle graphs usually give the percentage that each sector receives.
Expenditure of Major Industries
2000 ($3,087 million)
Legal
affairs, 12%
Research,
15%
Miscellaneous,
5%
Materials,
18%
Advertising,
12%
Construction,
2%
Taxes, 10%
Compensation,
26%
2010 ($4,851 million)
Research,
14%
Materials, 22%
Construction,
5%
Advertising,
15%
Legal
affairs, 4%
Taxes, 2%
Compensation,
38%
Q. The amount spent on materials in 2000 was 120% of the amount spent on
4
Pie Charts
Numerical Aptitude (Data I nterpretation) Pie Chart
2013 bankpoclerk.com
(1) research in 2000
(2) compensation in 2000
(3) advertising in 2010
(4) materials in 2010
(5) Data Inadequate
In 2000, 18% of the expenditures were for materials.
We want x where 120% of x = 18%; so x = 15%. Any
category that received 15% of 2000 expenditures gives
the correct answer, but only one of the five choices is
correct. Here, the answer is (1) since research received
15% of the expenditure in 2000.
Di recti ons : Study the fol l owi ng i nformati on to
answer the gi ven questions.
Percentage of students in various courses
(A, B, C, D, E, F) and Percentage of girals out of these.
Total Student: 1200
(800 girls + 400 boys)
Percentage in Various Courses
A
20%
B
15%
C
5%
D
35%
E
12%
F
13%
Total Girls : 800
Percentage of Girls in Couress
A, 30%
B, 10%
C, 2%
D, 30%
E, 14%
F, 14%
1. For which course i s the number of boys the
minimum?
(1) E (2) F
(3) C (4) A
(5) Data Inadequate
2. How many girls are there in course C?
(1) 44 (2) 16
(3) 40 (4) 160
(5) Data Inadequate
3. For course D what is the respective ratio of boys
and girls?
(1) 3 : 4 (2) 4 : 5
(3) 3 : 5 (4) 5 : 6
(5) Data Inadequate
4. For which pair of courses is the number of boys the
same?
(1) E and F (2) A and D
(3) C and F (4) B and D
(5) Data Inadequate
5. For course E, the number of girls is how much
percent more than the boys for course E?
(1) 250 (2) 350
(3) 150 (4) 80
(5) Data Inadequate
SOLUTION
Calculations for questions (15)
Courses Boys Girls Total
A 0 240 240
B 100 80 180
C 44 16 60
D 180 240 420
E 32 112 144
F 44 112 156
1. (4) Number of boys in course A is minimum i.e.,
zero.
2. (2) Number of girls in course C = 16
3. (1) Required ratio = 180 : 240 = 3 : 4
4. (3) Number of boys in each of courses C and F =44
5. (1) Required percentage =
112 32
32
100
=
18
32
100
=250
Numerical Aptitude Number System
For Any Guidance Call Our Expert at +91 8800734161

Click Here For Hard Copy of this Study Materials:
http://bankpoclerk.com/community/study-kit/ibps-po-capsule
Numerical Aptitude (Data I nterpretation) Bar Chart
2013 bankpoclerk.com
Quantities can be compared by the height or length of a bar in a bar graph. A bar graph can have either vertical
or horizontal bars. You can compare different quantities or the same quantity at different times.
DISABILITY BENEFICIARIES REPORTED AS REHABILITATED
(Number, as percent of all rehabilitated clients of state
vocational rehabilitation agencie, years 1995-2011)
20
15
10
5
0
Year 1985-95 1996 1997 1998 1999 2000 2001 1995-2005 2006 2007 2008 2009 2010 2011
Q. Between 2007 and 2011, the lowest number of disability beneficiaries were reported as rehabilitated in the
year
(1) 2007 (2) 2008
(3) 2009 (4) 2010
(5) None of these
The answer is (1) since the lowest bar is the bar for 2007.
Directions: Given here is a multiple bar diagram depicting the changes in the students strength of a college in four faculties
from 2000 2001 to 2002 2003. Study the diagram and answer the questions.
5
Bar Charts
Numerical Aptitude (Data I nterpretation) Bar Chart
2013 bankpoclerk.com

0
100
200
300
400
500
600
700
1990-91 1991-92 1992-93
S
t
u
d
e
n
t
s
t
r
e
n
g
t
h

Year
Arts
Science
Commerce
Law
2000-01 2001-02 2002-03
1. In which faculty was there a regular decrease in
students strength?
(1) Arts
(2) Science
(3) Commerce
(4) Law
(5) None of these
2. The percentage of students in Science faculty in
200001 was
(1) 26.9
(2) 27.8
(3) 29.6
(4) 30.2
(5) None of these
3. The total students strength in 200102 was how
many times that of Commerce students in the same
year?
(1) 3
(2) 4
(3) 5
(4) 6
(5) None of these
4. What was the percent increasein Science students
from the year 200001 to 200203?
(1) 50 (2)
2
66
3
(3) 75
(4) 150
(5) None of these
1. (1)
2. (3) Number of students in all faculties taken together
in 200001
= 600 + 400 +200 +150 =1350
Number of students in science faculty =400
Required percentage
400
100 29.6%
1350

3. (4) Total students strength in 199192
= 550 +500 +250 +200 =1500
Students strength in commerce in 200102 =250
Required solution

1500
6 times
250
4. (1) Students strength in Science in 200001 =400
Students strength in Science in 200203 =600
Increase = 600 400=200
Percent Increase

200
100 50%
400
Directions: The following bar chart shows the composition
of the GDP two countries (I ndia and South Africa).
Composition of GDP of Two Countries
Numerical Aptitude Number System
For Any Guidance Call Our Expert at +91 8800734161

Click Here For Hard Copy of this Study Materials:
http://bankpoclerk.com/community/study-kit/ibps-po-capsule
Numerical Aptitude (Data I nterpretation) Line Graphs
2013 bankpoclerk.com
Linegraphs are used to show how a quantity changes continuously. Very often the quantity is measured as time
changes. If the line goes up, the quantity is increasing; if the line goes down, the quantity is decreasing; if the line is
horizontal, the quantity is not changing. To measure the height of a point on the graph, use your note board as a
straight edge.
TRENDS IN INDUSTRIAL INVESTMENT, LABOUR PRODUCTIVITY,
EMPLOYMENT AND OUTPUT, 1980 TO 2007
(Index numbers, 1980 = 100)
1700
1600
1500
1400
1300
1200
1100
1000
900
800
700
600
300
200
100
0
400
300
200
100
0
1980 1985 1987 1989 1991 1993 1995 1997 1999 2001 2003 2005 2007
Note: The lefthand scale in roubles only to the
constant- price value of i ndustrial investment
Million roubles
Investment
Output
Productivity
Employment
Q. The ratio of productivity in 2007 to productivity in 1980 was about
(1) 1 : 4 (2) 1 : 3
(3) 3 : 1 (4) 4 : 1
(5) Data Inadequate
In 2007 productivity had an index number of 400, and the index numbers are based on 1980 =100. So the ratio
is 400 : 100 =4 : 1. Therefore, the answer is (4).
Directons: The graph given here shows a car following the linear path with uniform speed. Study the graph and answer the
questions.
6
Line Graphs
Numerical Aptitude (Data I nterpretation) Line Graphs
2013 bankpoclerk.com
1 2 3 4 5
30
24
18
12
6
Time (in hours)
1. The speed of the car is
(1) 12 km/ hr
(2) 6 km/ hr
(3) 18 km/ hr
(4) 24 km/ hr
2. The speed of the car in metres per minute is
(1) 60
(2) 100
(3) 600
(4) 1000
3. The distance travelled by the car in 4.5 hours is
(1) 27 km (2) 30 km
(3) 36 km (4) 40 km
1. (2) The car covers 6 km in 1 hour.
Speed =6 kmph
2. (2) Speed = 6 kmph

6 1000
60
m/ min. =100 m/
min.
3. (1) Required distance = Speed Time =6 4.5 =27
km
Directions: Study the following graphs carefully and answer
the questions given below:
Number of candidates appeared (In thousands) and
percentage of candidates qualified in a competitive
examination over the years are given below.
No. of Candidates appeared (In Thousands)
0
5
10
15
20
25
30
35
40
1997 1998 1999 2000 2001 2002 2003 2004
Years
Percentage of Candidates Qualified
0
5
10
15
20
25
30
35
40
45
1997 1998 1999 2000 2001 2002 2003 2004
Years
1. What was the ratio between the number of
candidates appeared in 1997 and the number of
candidates qualified in 2004?
(1) 14 : 5
(2) 5 : 14
(3) 3 : 7
(4) 7 : 3
(5) Data Inadequate
2. In which of the following years was the number
of candidates qualified the lowest among the given
years?
(1) 1997
(2) 2002
(3) 2001 (4) 1998
(5) Data Inadequate
3. What was the percentage drop in the number of
candidates appeared from the year 2002 to 2003?
Numerical Aptitude Number System
For Any Guidance Call Our Expert at +91 8800734161

Click Here For Hard Copy of this Study Materials:
http://bankpoclerk.com/community/study-kit/ibps-po-capsule
Numerical Aptitude (Data I nterpretation) Mix Diagram
2013 bankpoclerk.com
You can compare several categories by a graph of the cumulative type. These are usually bar or line graphs
where the height of the bar or line is divided up proportionately among different quantities.
Q. In 1986, roughly what percent of the federal prisoners received from the courts were guilty in other offenses?
(1) 10 (2) 15
(3) 25 (4) 30
(5) Data Inadequate
The total number of prisoners in 1986 was about 20,000, and guilty in other offences was about 5,000.
Since
5, 000 1
25%,
20, 000 4
the correct answer is (3).
In DI section of the question paper, the intention should be to attempt all questions as skipping them would
amount to losing precious scoring opportunities. There are two approaches to arrive at the solution. One is to work
on the data to arrive at the correct answer. The other one is the Elimination method which requires working
backwards by eliminating the wrong choices. Though the elimation method is more time consuming, it may still be
preferred where direct solution involves enormous calculation of data.
At times, examiners pose rather difficult data sets at the beginning of the sections. These are intended to be
SPEED BRAKERS which take away much of precious time. Therefore, as a rule, scan the whole section quickly
before actually attempting the questions and start with easier part of the section.
In some exams, data are presented in more than one table or graph. The objective is to test not only quantitative
7
Mix Diagrams
Numerical Aptitude (Data I nterpretation) Mix Diagram
2013 bankpoclerk.com
skills but also relational and analytical ability. Recently, in some exams the questions in this section are being
framed in caselet (paragraph) form, beginning with probability and reasoning questions. It is left to the reader to
study the case, call out requisite data and arrange it in a suitable form for meaningful interpretation. It is best to
arrange data with rough sketch to hasten comprehension.
I mportant Tips: Thesewill help in saving time, reducing mistakes and finding solution easily.
1. Get a general picture of the information by looking at the entire table or graph.
2. Read the table title, nomenclatures of columns and rows.
3. Simplify the questions being asked. Break down lengthy questions into smaller parts.
4. Use only the information given for finding solutions. Select the appropriate data for answering a question.
5. Eliminate impossible choices.
6. Avoid lengthy calculations.
7. Try to interpret through trends of the data in the graph. Whenever possible, try to answer the questions by
visualizing rather than by computing.
8. Where calculation is required prefer approximate values at the first stage. Go for exact calculation where
values are close and require exact answer.
9. Be careful to use proper units.
10. Make correct use of your knowledge of basic mathematical rules, principles and formulae.
11. Dont confuse in decimals and percent ages. For example, 0.5% =0.005.
12. Use pencil or straight edge of the answer sheet to read the graph and find approximate values.
13. Focus your answer on the question actually asked and not on what the question should be in your opinion.
14. Never do anything that is unnecessary.
15. Last, but not the least, make sure that the answer is sensible and reasonable.
Directions: Study the pie-chart given to answer these questions.
Composition of Exports and Imports
(per cent)
Exports (200809)
AAgriculture and Allied Products
BOres and Minerals
CManufactured Goods
Numerical Aptitude Number System
For Any Guidance Call Our Expert at +91 8800734161

Click Here For Hard Copy of this Study Materials:
http://bankpoclerk.com/community/study-kit/ibps-po-capsule
Numerical Aptitude (Data I nterpretation) Mix Diagram
2013 bankpoclerk.com
DCrude and Petroleum Products
ECapital Goods
I mports (20082009)
AFood and Allied Products
BFuel
CFertilisers
DPaper Board Manuf.and Newsprint
EOthers
FOther Bulk Items
GOthers
1. Indias trade balance during the year 200809 was:
(1) favourable (2) unfavourable
(3) neutral (4) Profitable
(5) Data Inadequate
2. Maximum export contribution has been due to :
(1) agriculture and allied products
(2) ores and minerals
(3) crude and petroleum products
(4) manufactured goods.
(5) Data Inadequate
3. The import of capital goods and other bulk items
incurred more thanof total import expenditure
(1) 50% (2) 60%
(3) 35% (4) 70%
(5) Data Inadequate
4. The imports exceeded by aboutmillions.
(1) US$ 8000 (2) US$ 8200
(3) US$ 8199.4 (4) US$ 8400
(5) Data Inadequate
5. The export of agriculture and allied products was
to the tune of US$ :
(1) 5700 millions (2) 6700 millions
(3) 4700 millions (4) 1730 millions
(5) Data Inadequate
6. The expenditure on fuel import compared to
contribution due to crude & petroleum products
export is:
(1) More (2) Less
(3) Equal (4) Unequal
(5) Inadequate data
Directions: Refer to the given line graph and the pie charts
and answer these questions:
FNIDistribution of Forest Land in North India FSIDistribution of
Forest Land in South India
7. How many hectares of FSI has been distributed
between 19942002?
(1) 3,00,500 (2) 2,77,500
(3) 6,57,000 (4) 9,00,000
(5) Data Inadequate
8. How many years witnessed a decline in FNI and
an increase in FSI?
(1) 2 (2) 3
(3) 4 (4) 5
(5) Data Inadequate
9. During 19942002 the greatest proportion of FNI
was put to commercial use in:
(1) 1994 (2) 1996
(3) 1999 (4) 2002
(5) Data Inadequate
Numerical Aptitude Number System
For Any Guidance Call Our Expert at +91 8800734161

Click Here For Hard Copy of this Study Materials:
http://bankpoclerk.com/community/study-kit/ibps-po-capsule



Click Here To View Study Kit
Study Kit For IBPS PO Examination
Click Here To Buy IBPS PO Study Kit
Study Kit For SBI PO/ Clerk Examination
Click Here To Buy SBI PO/Clerk Study Kit
Free Study Materials
Click Here For Study Materials
Book For IBPS,SBI & other Bank Examination
Click Here To Buy Books
Download Previous Years Exam Papers
Click Here To Download Previous Yeas Papers

You might also like